Você está na página 1de 32

Eluxidando a Física

Apresentação
O aprendizado de Física consiste em entender melhor o mundo que nos rodeia, bem como percebê-lo sob um
aspecto que vai além do que permite nossos sentidos. As fórmulas são muito mais do que uma expressão matemática
passíveis de aceitação e resignação do estudante, que têm um uso mecanizado em que se obtém como resposta um valor
numérico acabado. As fórmulas são uma consequência do fenômeno em questão, e que traduzem em uma linguagem
matemática aquilo que percebemos – adota-se, aqui, perceber no sentido de apreensão através dos sentidos – e que sem
essa ferramenta matemática, expressa-se de forma muito mais eloquente. A real investigação da Física consiste em entender
o que está por trás daquilo que se apreende, e o resultado se traduz como uma harmonia de grandezas físicas que trabalham
para produzir uma outra, dotada de sentido, e consequente do processo lógico da equação.

Mas, muito além do que saber aplicar as equações na Física, é importante compreender os fenômenos que regem a
natureza, não só por causa do vestibular, mas também para um conhecimento de mundo, aquisição de cultura e até um
pouco, uma pincelada, de conhecimento filosófico. Sim, a Filosofia está intrinsecamente ligada à Física, no entanto, aquela
tem uma capacidade de abrangência e de complexidade superiores a esta. Mas, perceba que, mesmo dentro da física, é
possível entendê-la de uma forma mais reflexiva e profunda do que o que se tem praticado e ensinado nas salas de aula.

Essa forma de abordagem pode parecer estranho, pelo fato de não ter sido mostrado dessa forma, ou até mesmo
desnecessária, visto que, para muitos estudantes, o estudo de Física só se deve ao fato de ser uma matéria que cobrada nos
vestibulares. No entanto, até mesmo os vestibulares vêm cobrando de modo mais intenso os conceitos em detrimento de
fórmulas memorizadas, ou, buscam mostrar nos exercícios formas diferentes – muitas vezes sequer visto em sala de aula –
de cobrar um conteúdo (tem aparecido algumas questões cujos tópicos são difíceis de identificar dentro da maneira
tradicional de separar as matérias; como Cinemática, Dinâmica, etc.). Esse livro prioriza justamente essa visão mais ampla,
mas que está longe de tornar o assunto mais complexo e/ou esgotar as mais diferentes formas de apresenta-lo. Este presente
livro visa o entendimento aliado à simplificação do conteúdo, dando lugar ao lado mais lógico e intuitivo do fenômeno, ao
invés de decorebas desnecessárias, que são muito úteis para aqueles que querem dar a matéria e se livrar logo do peso de
ter que explicar os porquês e de tirar dúvidas de alunos que têm interesse em entender mais.

Eu, não. Busco que o aluno entenda, ainda que eu pudesse só dizer “Decora que é mais fácil”, porque uma vez que
ele consegue internalizar o conteúdo, o conteúdo nunca mais será esquecido – talvez tenha que ser lembrados um ou outro
conceitos, mas a essência continuará na memória -, porque o conteúdo não foi só entendido, ele passou a fazer parte da
essência do estudante.

Espero, de fato, que este livro possa levar aos leitores a mesma alegria que tive quando sentia que entendia, ou
quando observava relações entre outras matérias, ou, ainda, quando passei a conseguir explicar fenômenos que não tinha
visto antes, mas que entendia porque aconteciam pelo fato de ter estudado Física. Obviamente, me falta muito o que
aprender, mas espero que, o que eu tenho para ensinar, possa servir de grande ajuda aos que têm dificuldade para entender
o conteúdo na sala de aula e aos que estejam sedentos por um conhecimento diferente na matéria.

Para dúvidas ou sugestões, mande e-mail para lucasgcunha.exatas@gmail.com.

Copyright © 2020 de Lucas Gosuen Cunha


Todos os direitos reservados. Este e-book ou qualquer parte dele não pode ser reproduzido ou usado de forma alguma sem
autorização expressa, por escrito, do autor ou editor, exceto pelo uso de citações breves em uma resenha do e-book.
Primeira edição, 2020.
Sumário

Cinemática ........................................................................................................................................................................ 5
Conceitos e Definições ..................................................................................................................................................... 5
Ponto material e corpo extenso.................................................................................................................................. 5
Grandeza........................................................................................................................................................................... 6
Espaço............................................................................................................................................................................... 6
Distância ....................................................................................................................................................................... 7
Operações com Vetores................................................................................................................................................... 7
Velocidade ........................................................................................................................................................................ 9
Velocidade média ........................................................................................................................................................ 9
Velocidade instantânea ............................................................................................................................................... 9
Velocidade relativa .................................................................................................................................................... 10
Velocidade vetorial .................................................................................................................................................... 10
Movimento Progressivo ............................................................................................................................................ 11
Movimento Retrógrado ............................................................................................................................................. 11
Equação do Movimento Uniforme (função horária) .................................................................................................. 11
Função genérica para a função horária ................................................................................................................... 11
Exemplos aplicando função horária......................................................................................................................... 11
Análise gráfica sxt...................................................................................................................................................... 12
Função horária para corpos extensos ...................................................................................................................... 13
Tipos de movimento no gráfico ................................................................................................................................ 13
Aceleração vetorial .................................................................................................................................................... 14
Aceleração centrípeta ............................................................................................................................................... 15
Movimento Uniformemente Variado (MRUV ou MUV) ............................................................................................... 15
Movimento acelerado................................................................................................................................................ 15
Movimento retardado ............................................................................................................................................... 15
Classificação dos movimentos no gráfico (velocidade x tempo) ........................................................................... 15
Cálculo da Velocidade e Espaço no Gráfico................................................................................................................... 16
Equações do MUV........................................................................................................................................................... 17
BÔNUS: Equação do primeiro e do segundo grau ........................................................................................................ 18
Movimento Circular Uniforme (MCU) .......................................................................................................................... 20
Espaço Angular .......................................................................................................................................................... 21
Velocidade Angular ................................................................................................................................................... 21
Elementos rotativos acoplados ................................................................................................................................ 22
Mecânica da Bicicleta ................................................................................................................................................ 22
Movimento Circular Uniformemente Variado(MCUV) ................................................................................................ 23
Lançamento Oblíquo ...................................................................................................................................................... 24
Eluxidando a Física

Equações do movimento na horizontal: .................................................................................................................. 25


Equações do movimento na vertical: ....................................................................................................................... 25
Exercícios ....................................................................................................................................................................... 27
Gabarito .......................................................................................................................................................................... 32

4
Eluxidando a Física

Cinemática Novamente, lembre-se de essa consideração é


A cinemática é o ramo da física que trata dos válida sempre levando em conta a que está fazendo a
movimentos dos corpos sem levar em conta a causa desse comparação. Já que eu coloquei imagens de corpos em
movimento, isto é, não importa se o corpo em estudo está escala macroscópica. Vamos ver imagens em que um fio
se movimentando devido à força da gravidade ou se de cabelo pareça ser gigante!
alguém está empurrando. O interesse reside
exclusivamente em olhar o movimento, somente.
E como estudamos esse movimento?
Fazemos uso de funções e equações e, com estas,
verificamos qual é o comportamento do corpo a partir de
parâmetros como espaço, velocidade e aceleração, que
serão definidas adiante.

Conceitos e Definições
Antes de se iniciar o estudo do movimento com as
equações é necessário, antes, ter familiaridade e um
fortalecido conhecimento dos conceitos básicos1.
Ponto material e corpo extenso
Ponto material é o corpo cujas dimensões físicas,
isto é, comprimento, largura e altura não precisam ser
levados em conta no meio em que se encontram. Isso não
significa que o corpo não tenha dimensões, apenas que
estas dimensões físicas são desprezíveis levando em
conta aonde estão. Isso significa que a afirmação “um trem
de carga é grande!” não necessariamente é verdadeira. A
pergunta a se fazer é: grande em relação a que? Se
comparar com o tamanho do planeta Terra, ele é
desprezível, logo, pode ser considerado um ponto
material em relação à Terra.
“Ah, então eu posso afirmar sempre que o planeta
Terra é grande sem que ela seja um ponto material!”,
Figura 2: olha a comparação de uma célula à esquerda em relação a
pode-se questionar você. Hmm, eu diria que não. espessura da fibra de seda. Agora compare a largura do cabelo humano
Só para te dar uma melhor ideia: o Sol é uma estrela com ela. Incrível, não? Você pode ver mais no site
com o raio 109 vezes maior do que o da Terra, o que http://htwins.net/scale2/ de forma interativa!
corresponde a um volume 1.300.000 (um milhão e
trezentos mil!) vezes maior, aproximadamente. Olhe na Acredito que eu tenha conseguido te convencer
figura abaixo o tamanho do Sol em comparação com a que tamanho é relativo! Aliás, não só o tamanho. O
estrela chamada UY Scuti. movimento também o é! Confira a seguir.
Bom, acho que já deu pra perceber que tamanho
depende do que você está comparando. Repouso e Movimento
O movimento de um corpo pode ser
caracterizado quando há um distanciamento ou uma
aproximação, conforme o tempo corre, do corpo em
relação a um referencial. Repare que, assim como para
saber se o corpo é ponto material ou corpo extenso,
precisamos de alguém para comparar. Se esse alguém está
se distanciando ou se aproximando, isso significa que há
movimento em relação a esse alguém (Figura 3).
O repouso também leva em conta um referencial,
um meio de comparação. Entretanto, o repouso é
designado quando não há distanciamento ou
aproximação do corpo com o referencial com o tempo.

Lembre-se:
Figura 1: Tamanho do Sol em relação à estrela UY Scuti. Você está O movimento e o repouso são relativos!
conseguindo enxergar? Se sim, parabéns, imagino que é capaz de ver um
átomo a olho nu. (fonte: https://telescopeobserver.com/uy-scuti/)
Para ficar mais claro, veja o exemplo seguinte:
Corpo extenso é o corpo em que tem que ter suas quando você está dentro do ônibus, os outros passageiros
dimensões físicas levadas em conta dado o meio em que se mantêm na sua proximidade ao longo de todo o trajeto,
se encontram. É exatamente o oposto do que o ponto isto é, eles não se deslocaram para um lugar mais longe ou
material representa. mais próximo de você (supondo que todos ficaram

1 Embora comumente deixados de lado, os conceitos básicos são como sinônimos. Portanto, atente-se aos conceitos antes de seguir para
fundamentais para entender rigorosamente diferenças de termos como o lado mais prático.
distância e deslocamento, que muitas vezes são tomados, erroneamente,

5
Eluxidando a Física

sentados ao longo de todo o trajeto). Com isso, pode-se Escalar é a grandeza que é satisfeita somente
dizer que os passageiros ficaram em repouso em relação quando o valor é fornecido. Por exemplo, “um corpo de 2
a você. Agora, se fizermos a consideração levando em kg”. Apesar de não saber de que corpo se trata, o sentido
conta da sua distância em relação a uma árvore que se da grandeza massa em questão é totalmente
localiza no ponto que ônibus saiu sabemos que no final do compreendido. Outras grandezas escalares: temperatura
trajeto essa árvore não está mais ao alcance da sua vista (“35°C”, já faz compreender a totalidade dessa grandeza),
devido ao deslocamento. Portanto, a árvore esteve em área, volume, etc.
movimento. Repare que nos dois casos citados o Vetorial é a grandeza que não basta dizer
referencial é você, mas dependendo do objeto em questão, somente o seu valor (também chamado de módulo), como
pode haver ou não movimento. Logo, diz-se que o no escalar, mas também é necessário dizer sua direção e
movimento é relativo, isto é, depende da relação entre o sentido para o qual essa grandeza se dirige. Por exemplo,
referencial e o objeto. "ontem eu estava passando pelo cruzamento a 40 km/h".
Ao ouvir essa frase, não está claro em qual das duas ruas
do cruzamento o motorista está (direção) e, uma vez
estabelecido qual direção, pergunta-se se estava indo ou
vindo (sentido). Para isso, são três informações
necessárias para uma completa compreensão de uma
grandeza vetorial: módulo, direção e sentido.
A direção e o sentido podem ser do seguinte
modo:
 Direção horizontal, sentido esquerdo ou direito
 Direção vertical, sentido para cima ou para baixo
Figura 3: repare como o carro está em movimento em relação à rua e aos
 Direção oblíqua (inclinado), sentido para cima ou para
objetos em torno dela. A câmera não consegue focar pois ou o foco fica baixo
no veículo ou na rua. É comum em alguns casos querer representar
somente o valor da grandeza vetorial. A preocupação se
reside unicamente no número e não na direção e sentido
em que se aponta. Para esses casos, fala-se em módulo do
vetor. Por exemplo, “o módulo do vetor velocidade em que
eu passava pelo cruzamento é de 40 km/h”. Nesse caso, o
sentido da frase está completo, pois foi deixado claro que
o importante é somente o módulo.

Figura 4: Veja que há foco em toda a imagem dentro do ônibus. Isso se


deve ao fato de que não há movimento relativo entre as partes da Figura 5: Representação de vetores velocidade. Repare que uma
imagem que estão dentro do ônibus. velocidade de 10 m/s pode se dar em diferentes direções. Para que os
vetores estejam representados completamente, deve-se fornecer
direção e sentido.
Grandeza
Grandeza se refere a tudo que se pode mensurar,
isto é, medir. Podemos dizer, então, que distância é uma Espaço
grandeza, pois ela mensura o tamanho. A velocidade É o ponto no qual o corpo está localizado. Tem
também é uma grandeza porque mensura o quão rápido ideia semelhante a uma coordenada geográfica. Se numa
um corpo se movimenta, assim como a força, que mede os reta você está em cima do ponto 4 metros, em que o zero
diversos níveis de intensidade. Para saber com o que vai é um referencial arbitrário qualquer (também chamado
ser medida a grandeza, são necessários padrões de de origem dos espaços, isto é, o ponto 0 metro), diz-se que
medidas que façam isso, que são as unidades. Estas o móvel se encontra no espaço 𝑠1 =4m. Se o móvel se
fornecem valores numéricos seguidos de padrões desloca e passa a ocupar o ponto distante de 7m do
estabelecidos para se saber como é essa medida. Por referencial (ou seja, 7 metros distante do ponto em que
exemplo, foi padronizada a medida quilograma 2, cuja está o 0 metro), então o espaço o qual o móvel se localiza
definição tem que ser de algo externo; não pode definir é 𝑠2 =7m. Com isso, passa a ser possível calcular qual foi a
quilograma como metade de 2 quilogramas, pois isso é variação de espaço a partir do referencial dado. O símbolo
uma definição circular. É necessário ter o padrão para matemático que representa a variação é a letra grega
fazer qualquer outra medida; por exemplo: um corpo de 3 delta (∆).
kg é a mesma coisa que 3 × 1 kg, isto é, o corpo tem uma Nesse exemplo ilustrado, a variação do espaço foi
massa 3 vezes maior do que a massa padrão. de 3m, pois o espaço final era s2 = 7m e o espaço inicial
As grandezas podem ser divididas em escalar e era s1 = 4m. Matematicamente, representamos da
vetorial. seguinte maneira: ∆s = s2 − s1 = 7 − 4 = 3m. Repare

2 A massa era definida a partir de um cilindro metálico de uma liga de


platina e irídio de 39 mm de diâmetro e 39 mm de altura. Essa definição
foi substituída em 2019, sendo escrita a partir da constante de Planck.

6
Eluxidando a Física

que o termo variação diz respeito ao espaço final menos o


inicial.

Dica!
É importante ficar atento quanto à informação de um
exercício quanto ao espaço: se se trata do espaço ou
na variação do espaço

Numa rodovia, se há uma placa escrito km 10, isso não


significa que você andou 10 km, mas sim, que é um ponto
do espaço que está a 10 km do marco zero. Ir do km 10
para o km 20 corresponde a uma distância de 10 km
andados Figura 7: o vetor muda de sentido quando o espaço inicial e final
trocam
Deslocamento (r⃗)
A variação do espaço do qual estava sendo falado Distância
acima é precisamente o deslocamento. Portanto, o A distância é bastante semelhante ao
deslocamento do corpo no exemplo acima foi de 3m, que deslocamento, entretanto não é vetorial, é escalar, isto é,
é o espaço final (s2 = 7m) menos o espaço inicial (s1 = não tem direção ou sentido, é somente um valor
4m). Se o corpo fizesse um movimento com mais paradas numérico.
do tipo: sai do espaço 4m, se desloca até o 5m e depois A distância, diferente do deslocamento que só
para novamente, vai até o 6m e para e, por fim, vai até o interessa o espaço final e o inicial, leva em conta todo o
7m e para novamente. Esse caso, agora envolvendo os trajeto percorrido. A ilustração abaixo mostrará
espaços 4m e 5m, apesar de ser mais extenso, possui visualmente uma diferença clara entre distância e
exatamente o mesmo deslocamento do caso anterior, pois deslocamento.
o que interessa é somente o espaço final e o inicial, e não
os intermediários. Em outras palavras, o deslocamento
leva em conta somente os pontos de saída e de chegada, e
é definido por um vetor 3 que sai do ponto inicial e vai até
o ponto de chegada da trajetória.

Figura 8: Um corpo que se encontra inicialmente em 1 se


desloca verticalmente até 2 percorrendo 3 metros e, em
seguida, se desloca horizontalmente de 2 até 3,
percorrendo 4 metros. A distância total percorrida foi de
Figura 6: o deslocamento continua o mesmo ainda que o corpo realize 7 metros, enquanto o deslocamento foi de 5 metros.
paradas intermediárias.
Suponha um circuito de corrida em que cada volta na
Até agora analisamos somente o valor, isto é, o pista tenha 10 km de extensão. Ao final de 10 voltas,
módulo do deslocamento. Uma vez que o corpo saiu do pede-se:
espaço 4m e foi até o 7m o deslocamento foi de +3m, o a) Distância;
que resulta em um valor diferente se ele saísse dos 7m e b) Deslocamento4.
chegasse nos 4m, levando a um deslocamento de -3m
(duas grandezas vetoriais com sinais opostos possuem
Operações com Vetores
direções contrárias). Portanto, o deslocamento é uma
Conforme vimos no tópico sobre deslocamento,
grandeza vetorial.
algumas grandezas físicas são vetoriais. E, diferente da
soma escalar (soma comum, a qual estamos acostumados
Lembre-se:
quando somamos números), na soma vetorial não basta
Vetores não podem ser somados igual escalares somar numericamente os valores dos vetores para obter
o valor resultante. Para somar dois vetores é necessário
que se tenha a orientação deles e o valor de cada um para
que se possa fazer a operação.
Antes de mostrar como fazer os cálculos com
vetores, será visto como encontrar a orientação do vetor

3 Um vetor é representado por uma seta em linha reta que define a 4A distância será de 100 km percorridos, enquanto o deslocamento
direção e o sentido dado pelos estado final e estado inicial será de 0, já que ao final das 10 voltas a posição do carro (posição final)
é a mesma que a posição inicial.

7
Eluxidando a Física

resultante. Uma das maneiras de somar vetor (e a mais


fácil), é juntar a parte traseira de um vetor com a parte
frontal do outro. Por exemplo:

Se a operação num vetor for de multiplicação com


um escalar, basta aumentar o valor do vetor pelo número
Figura 9: Os vetores foram organizados de forma a formar uma fila. Não
precisa ter ordem, qualquer vetor pode ser alinhado a qualquer outro. O que está sendo multiplicado. Se multiplicar o vetor por 3,
importante é que sejam alinhados. seu comprimento fica 3 vezes maior. Mas se for
multiplicado por -3, além de multiplicar por 3 vezes o
Uma vez alinhados, para descobrir o vetor tamanho, seu sentido se inverterá. Quando a operação for
resultante dessa operação fica muito simples. O vetor de divisão, o procedimento é o mesmo, uma vez que a
resultante “sai da saída e chega na chegada”. Visualmente: divisão é a multiplicação do inverso de um número
(dividir por 2 é o mesmo que multiplicar por ½).
Mas, se for pedido para calcular
quantitativamente, isto é, numericamente a soma de dois
vetores? Vejamos como proceder com um exemplo:

Figura 10: O vetor resultante é dado pelo vetor em vermelho

Portanto, para calcular a orientação do vetor


resultante, pode ser válido lembrar que ele “sai na saída e
chega na chegada”, isto é, sai da parte traseira do primeiro
e chega até a parte frontal do último. Para o cálculo do vetor resultante, pode
Agora suponha que queira fazer a operação A ⃗⃗ − B
⃗⃗ decompor as componentes dos vetores em x e y, e em
seguida, somar os valores em x com os valores em y e, em
os vetores abaixo:
seguida, determinar o valor do vetor resultante com
Pitágoras:
√3
Ax = A. cos(30°) = 2. = √3
2
1 1
Ay = A. sen(30°) = 2. =
2 2

√3 √3
Bx = B. cos(30°) = 1. =
2 2
Para facilitar a operação, antes, é necessário 1 1
ordenar em fila os dois vetores. Mas note que não é para By = B. sen(30°) = 1. =
2 2
somar B e sim –B. Para isso, basta inverter o sentido de B,
e depois enfileirá-lo com A. √3 3√3
Cx = Ax + Bx = √3 + =
2 2
1 1
Cy = A y + By = + = 1
2 2
3√3 2 √22
C = √Cx2 + Cy2 = √( ) + 1² =
2 2
Para descobrir o ângulo, basta fazer:
𝐶𝑦
𝜃 = tan( )
𝐶𝑥
Apesar de ser eficaz, isto é, resolve o exercício, há
Como A-B=A+(-B), basta fazer o mesmo outro método que é bem melhor e mais rápido. É a Regra
procedimento: “sai na saída, chega na chegada”. do Paralelogramo.

8
Eluxidando a Física

Essa regra consiste em duplicar os dois vetores e necessariamente a origem dos espaços, mas um ponto de
traçá-los paralelamente, conforme o desenho abaixo: partida qualquer do móvel, assim como o cronômetro não
precisa iniciar quando este está zerado (mas o caso mais
comum é o cronômetro iniciar no zero).
É importante ressaltar que a velocidade média
não significa que o móvel tenha sempre a mesma
velocidade. Sua velocidade pode variar o trajeto inteiro. O
que a velocidade média nos diz é: se o móvel tivesse feito
o trajeto que fez a uma velocidade constante, esta
velocidade seria a velocidade média.
Exemplo 1: Um móvel sai do espaço 3 metros, de
modo que o cronômetro inicia a contagem do tempo aos 4
segundos, devido ao descuido de não tê-lo zerado antes de
cronometrar novamente, e percorre o trajeto até que o
Agora, para calcular o vetor resultante A+B, móvel chegue ao espaço de 8 metros. Ao chegar nesse
basta fazer lei dos cossenos no triângulo de cima: ponto o cronômetro é parado no tempo de 5 segundos.
Calcule a velocidade média.
Resposta:
∆s s2 − s1 8 − 3 5
vm = = = = = 5 m/s
∆t t 2 − t1 5 − 4 1
Obs. 1: muita atenção para que o tempo 𝑡2 fique
embaixo do espaço 𝑠2 e o tempo 𝑡1 fique embaixo do
espaço 𝑠1 na fração.
Obs. 2: repare que na equação é variação de
espaço (representado no SI por m) dividido pelo tempo
(representado no SI por s). Por isso a velocidade é
representada como m/s. Observe que a própria equação
nos fornece qual unidade é pertencente à grandeza
Obs. 3: Esse tipo de exemplo é bem atípico. O mais
comum é que o móvel saia da origem dos espaços com o
cronômetro zerado. Mas se houve entendimento dessa
Obs: o valor de 75° é o ângulo formado entre os vetores explicação acima, então os casos mais simples serão
A e B, 45°+30°=75°. entendidos com facilidade.

Lei dos cossenos: C 2 = A2 + B 2 − Exemplo 2: A distância entre das cidades A e B é


2. A. B. cos(105°) de 120 km e das cidades B e C é de 60 km, sendo A, B e C
Agora, se multiplicar um vetor por um valor colineares e sequenciais. Suponha que ao longo dessa
escalar, sua direção e sentido não mudam, apenas o seu rodovia, o fluxo do trânsito era normal e o carro
módulo, seja aumentando-o, diminuindo-o ou trocando o manteve-se ao longo do trajeto AB a uma velocidade de
sentido: 120 km/h, mas, devido a uma manutenção na estrada, o
trecho BC teve lentidão, e o motorista precisou manter-
se a 30 km/h. Qual foi a velocidade média do trecho AC?

Resposta:
Velocidade ∆s1 ∆s1 120 km km h
A velocidade, no seu sentido mais geral, mede a vm 1 = → ∆t1 = = =1 = 1 km.
∆t1 vm1 120 km km km
variação do espaço em função do tempo ou, em outras h h
palavras, mede o ritmo da variação do espaço. = 1h
Obviamente, quanto maior esse ritmo, maior a variação ∆s2 ∆s2 60 km km h
do espaço no tempo e, portanto, maior a velocidade. vm 2 = → ∆t 2 = = =2 = 2 km.
∆t 2 vm2 30 km km km
Repare que anteriormente foi abordado sobre a h h
variação do espaço, mas nada foi falado sobre o tempo. A = 2h
velocidade é justamente a introdução do tempo, é a ∆st 180 km
variação do espaço em relação à variação do tempo. vm t = = = 𝟔𝟎 𝐤𝐦/𝐡
∆t t 3h
Matematicamente:
∆𝐬
𝐯= (𝟏) Velocidade instantânea
∆𝐭
Já a velocidade instantânea diz respeito à
Velocidade média velocidade em um dado instante de tempo. No instante
O cálculo da velocidade média é igual a equação seguinte (com instante me refiro a um tempo
acima, em que o tempo é iniciado a partir do momento em minimamente pequeno), a velocidade instantânea pode
que o móvel (corpo que se move) sai do ponto inicial, do ser outra. É a velocidade que é indicada no velocímetro
seu ponto de saída, e a contagem do tempo só é parada dos carros.
quando o móvel chega no seu ponto final. É importante
ressaltar que o ponto de partida do móvel não precisa ser

9
Eluxidando a Física

b) está contra o movimento da esteira;


Resposta:
a) Quando a pessoa e a esteira rolante se
movimentam no mesmo sentido, as velocidades
se somam:
vpessoa,chao = vpessoa,esteira + vesteira,chao
vpessoa,chao = 3 + 4 = 7 m/s
Ou seja, a pessoa anda mais rápido do que se
estivesse andando fora da esteira, uma vez que a esteira
ajuda por estar no mesmo sentido.
b) Quando a pessoa e a esteira se movimento em
sentido contrário, as velocidades se subtraem:
Figura 11: velocímetro marcando a velocidade instantânea.
vpessoa,chao = −vpessoa,esteira + vesteira,chao
Num caso em que um carro se mantém a uma vpessoa,chao = −4 + 3 = −1 m/s
mesma velocidade ao longo de todo o trecho, a velocidade A pessoa por ter uma velocidade levemente
média e a velocidade instantânea se coincidem, pois não superior à da esteira, tem movimento no sentido negativo.
importa o instante, a velocidade é sempre a mesma. Se a pessoa e a esteira tivessem a mesma velocidade, a
Exemplo 3: A partir do gráfico v x t (velocidade x pessoa ficaria parada em relação ao chão.
tempo) seguinte, indique qual a velocidade do móvel nos Velocidade vetorial
instantes 2s, 3s e 5s. Assim como a velocidade escalar diz respeito à
variação escalar do espaço, a velocidade vetorial diz
respeito à variação do deslocamento (vetor que define a
diferença do espaço final e inicial).
r⃗
⃗⃗ =
v (𝟐)
∆t
Se o que quiser saber for somente o módulo da
velocidade vetorial, isto é, o valor da velocidade vetorial,
sem levar em conta a direção e sentido, a representação
do módulo pode ser uma das duas seguintes maneiras:
|r⃗| r
|v
⃗⃗| = ou v =
Δt Δt
Exemplo 5: Um barco possui a velocidade em relação ao
Resposta: como o eixo vertical já informa rio de 12 m/s e o rio possui uma correnteza de 5 m/s. A
sobre a velocidade é fácil enxergar as velocidades distância entre as margens do rio é de 120 m.
v(2) = 4m/s a) Quanto tempo o barco demora para chegar até o outro
v(3) = 4m/s lado da margem?
v(5) = 0 b) Qual a distância total percorrida pelo barco em relação
No tópico de exercícios, o E1 é um exercício de à margem?
análise gráfica que leva em conta o s x t (espaço x tempo).
Para uma boa compreensão dessa matéria é fundamental Resposta: O barco e o rio estão esquematizados abaixo:
que esse exercício seja realizado.

Velocidade relativa
A velocidade relativa é usada quando estamos
falando da velocidade entre dois móveis. Suponha um
carro A no mesmo sentido de um carro B. Se o carro A tem
uma velocidade de 50 km/h e o carro B está a uma
velocidade de 80 km/h, então a velocidade relativa entre
os carros é de 30 km/h.
vBA = vB − vA = 80 − 50 = 30 km/h
Agora, num segundo caso, suponha que esses
dois mesmos carros estão na mesma velocidade, porém
no sentido contrário. A velocidade relativa entre eles é
somada, portanto, 130 km/h. Como os sentidos são
contrários, um terá velocidade positiva e o outro,
negativa. a) Note que independentemente da velocidade da
vBA = vB − vA = 80 − (−50) = 80 + 50 = 130 km/h correnteza, o
barco alcança a outra margem, pois a correnteza não tem
Exemplo 4: Uma pessoa está andando sobre uma esteira velocidade na direção do barco, nem o barco tem
rolante com uma velocidade de 4 m/s em relação a ela, velocidade na direção da correnteza.
enquanto a esteira possui a velocidade de 3 m/s em Logo, o tempo de travessia é simplesmente:
d 120
relação ao chão. Calcule a velocidade da pessoa em vb = → 12 = → 𝐭 = 𝟏𝟎 𝐬
relação ao chão quando: ∆t t
a) está a favor do movimento da esteira;

10
Eluxidando a Física

b) Sempre que a referência for a margem deve ser s(t) = 3t + 2


considerada a t = 0s → s(0) = 3.0 + 2 = 2m
velocidade relativa entre o barco e o rio. Quando tomado t = 1s → s(1) = 3.1 + 2 = 5m
em relação à margem, o barco tem uma velocidade de 12 t = 2s → s(2) = 3.2 + 2 = 8m
m/s na vertical e 5 m/s na horizontal. t = 3s → s(3) = 3.3 + 2 = 11m
Nestes 10 segundos, o barco percorreu na
horizontal. Repare a cada segundo que passa, o espaço varia
dx em 3 metros, isso dá 3 metros a cada segundo, ou seja, a
vx = → dx = vx . ∆t = 5.10 = 50 m
∆t velocidade é de 3 m/s. Outro modo de calculá-la é pegar
quaisquer dois espaços e dividir pelo tempo.
Na vertical, percorreu
dy Função genérica para a função horária
vy = → dy = vy . ∆t = 12.10 = 120 m
∆t ∆s
(O valor de 120 m era óbvio de ser encontrado, uma vez vm = → ∆s = vm . ∆t → sf − si = v. t → sf = si + v. t
∆t
que foi exatamente a distância que levamos em 𝐬(𝐭) = 𝐬𝟎 + 𝐯. 𝐭 (𝟑)
consideração no item anterior).
Obs.: A velocidade média é a própria velocidade
instantânea pelo fato de que no movimento uniforme a
velocidade nunca varia.
A função horária de qualquer movimento pode
ser descoberta a partir da equação (2). Veja que o termo
s0 no exemplo acima era de 2m (conforme a primeira
linha), mas isso poderia ser visto diretamente na equação,
bem como a velocidade. Basta alinhar a função horária
fornecida no exercício com a equação (3).
s(t) = 3t + 2
s(t) = v. t + s0
Portanto: v = 3m/s e s0 = 2m
Vamos analisar a função horária do MRU,
equação (2), a partir de uma outra visão. Note que s(t) é o
Portanto, para encontrar a distância, basta fazer do espaço final, s0 é o termo do espaço inicial, e v.t é o
um Teorema de Pitágoras: termo responsável pela variação que levou o corpo ir do
d2 = 502 + 120² espaço inicial até o espaço final. Portanto, podemos
𝐝 = 𝟏𝟑𝟎 𝐦 elaborar uma frase bem intuitiva a respeito da equação
(2): “o ponto final que eu cheguei é o ponto que saio mais
Obs.: Esse é o triângulo (5x, 12x, 13x). Aparece o que eu andei”, ou de maneira mais geral, “o fim é igual
com muita frequência nos exercícios. ao início mais o que andou”. Haverá outras equações na
física em que se podem se utilizar a mesma frase nessa
Movimento retilíneo uniforme (MRU ou MU) forma parecida com saldo de banco “o que eu tenho no
Como o próprio nome descreve, o MRU se trata de banco é o quanto eu tinha mais/menos o quanto eu
movimentos em que a velocidade não varia (observe que coloquei/retirei”.
o espaço, sim, varia) e seguem linha reta. Essa é uma
condição para que a função horária (função que relaciona
o tempo em função do espaço) fique descrita por uma
função do primeiro grau.
Movimento Progressivo Figura 12: representação visual da função horária representada pela
equação (2)
Movimento caracterizado pelo corpo que possui
velocidade maior que zero (positiva). O móvel avança no
sentido positivo em relação ao arbitrado, isto é, no sentido Exemplos aplicando função horária
de crescimento do eixo do espaço. Lembrar que Exemplo 6: um móvel 1 com uma velocidade de
progressivo remete à ideia de progredir, ou seja, avançar 20 m/s no sentido positivo do eixo dos espaços parte 100
no sentido positivo. metros à frente de um móvel 2 que está a uma velocidade
de 30 m/s. Considerando que ambos percorrem o mesmo
Movimento Retrógrado
sentido do eixo. Em quanto tempo os móveis se
Velocidade menor que zero (negativa). O móvel
encontrarão? E em qual ponto do eixo dos espaços?
avança no sentido negativo em relação ao arbitrado, isto
Resposta:
é, no sentido de decrescimento do eixo do espaço.
Retrógrado lembra de regredir, ou seja, voltar em relação
ao sentido positivo. Logo, ficar cada vez mais negativo.
Outra maneira de ver é que o espaço varia negativamente,
ou seja, aponta para posições cada vez menores
Resolução 1: É de elevada importância que em
um exercício a situação descrita no enunciado seja
Equação do Movimento Uniforme (função horária)
colocada na forma de desenho. Não só para facilitar
A função horária é a função que relaciona o tempo
visualmente o exercício, mas também para que não seja
e o espaço percorrido pelo móvel. Veja, a seguir, um
mais necessário ler o enunciado toda vez que tiver alguma
exemplo de função horária:

11
Eluxidando a Física

dúvida. Então vamos fazer a função horária de cada


móvel:
Móvel 1: v1 = 20 m/s e s01 = 100m
s(t) = s0 + v. t → s1 (t) = 100 + 20. t

Móvel 2: v2 = 30 m/s e s02 = 0


s(t) = s0 + v. t → s2 (t) = 0 + 30. t → s2 (t) = 30. t

O exercício pede o tempo em que os móveis se


encontrarão. Sabemos que vão se encontrar quando
estiverem no mesmo espaço do eixo dos espaços.
Portanto:
s1 (t) = s2 (t)
Agora, substituindo as funções horárias Figura 13: comparação entre a função horária de dois móveis 1 e 2,
genéricos.
encontradas acima e resolvendo:
100 + 20. t = 30. t
Como a função horária é uma função do primeiro
100 = 30. t − 20. t
grau, isto é, x tem grau 1 (x está elevado a 1), a função
100 = 10. t
gráfica será uma reta. Como a partir do gráfico podemos
10 = t
saber as velocidades dos móveis? Nesse gráfico
𝐭 = 𝟏𝟎𝐬
desenhado, não tem como saber as velocidades, mas dá
Portanto, levará 10 segundos para que os móveis
para saber que o móvel 2 é mais rápido. Como? Pelo fato
se encontrem.
de que para um mesmo intervalo de tempo, a variação do
espaço de 2 é maior do que a variação do espaço de 1, isso
Resolução 2: usando a velocidade relativa.
significa que ele tem maior velocidade. Em outras
Carro 1: v1 = 20 m/s e s01 = 100 m palavras, a velocidade indica a inclinação da reta.
Carro 2: v2 = 30 m/s e s01 = 0 m No tópico de geometria analítica em matemática,
Com velocidade relativa: na equação da reta o termo que multiplica a variável (a
Carro 1: v1 = 0 m/s e s01 = 100 m variável é o tempo t na função horária) é denominado
Carro 2: v2 = 10 m/s e s01 = 0 m coeficiente angular que, na função horária, é justamente a
Logo, o tempo para o segundo carro encontrar o velocidade. O termo independente, isto é, o termo que não
primeiro é: está multiplicado pelo tempo, é denominado coeficiente
d d 100 linear. Quanto maior o coeficiente linear, para um mesmo
v1 = → ∆t = = = 10 s
∆t v1 10 coeficiente angular (mesma inclinação), mais deslocado
para cima é a reta (Figura 11).
Agora, para responder a segunda pergunta do
exercício, isto é, em qual ponto os móveis se encontrarão,
basta substituir t=10s em quaisquer uma das equações
(já que para t=10s há o encontro, ambos estarão no
mesmo ponto do eixo dos espaços)
Como a função horária do móvel 2 é mais simples,
a substituição será feita nele (poderia ser no móvel 1
também):
s2 (t) = 30. t → s2 (10) = 30.10 = 300m
Os móveis se encontrarão no ponto +300 metros.
Isso significa que o móvel 1 avançou 200 metros (pois
partiu do ponto 100 metros) e o móvel 2 avançou 300
metros. Figura 14: comparação entre os coeficientes lineares (termos
independentes que, nesse caso, são os espaços iniciais dos móveis) das
Análise gráfica sxt retas 1 e 2. Nesse caso, ambos os coeficientes lineares são positivos, pois
estão sobre a parte positiva do eixo y (eixo dos espaços s).
Agora vamos analisar como fica o gráfico de uma
função horária do tipo da equação (2), comparando dois
Veja que no caso da Figura 11 os dois móveis
móveis.
nunca se encontrarão, pois estão distantes entre si
Supondo s01 = s02 > 0 e v2 > v1 > 0, o gráfico
sempre com a mesma velocidade. Por isso as retas são
ficará do seguinte modo:
paralelas, devido ao fato das velocidades serem iguais.
Agora, no caso da velocidade de um móvel ser
Dica! negativa, a inclinação da reta ficará para baixo e quanto
O número que multiplica o termo x na função horária mais negativa for a velocidade, mais inclinado para baixo
representa a inclinação do gráfico ficará.

Dica!
O termo independente é o que determina a posição
inicial

12
Eluxidando a Física

Função horária para corpos extensos retrógrado, basta que a inclinação seja para baixo, em um
Como foi citado no início nas definições, corpo gráfico s x t. Já no gráfico v x t, se a reta horizontal estiver
extenso é aquele corpo cujas dimensões devem ser no lado negativo da velocidade, então o movimento é
levadas em conta. Exercícios muito comuns de aparecer retrógrado.
falam sobre ultrapassagens de um trem em relação ao Resumindo, os movimentos nos gráficos são:
outro. Para saber quando ocorre ultrapassagem, é
necessário saber a extensão do corpo.
Para dois pontos materiais a ultrapassagem
ocorre no momento em que um corpo alcança o outro.
Mas no caso do corpo extenso quando um alcança o outro,
a ultrapassagem é iniciada e só finaliza quando todo o
corpo ultrapassou o outro.

Dica!
Num gráfico s × t:
Inclinação>0: v>0
Inclinação<0: v<0

Encontro e Paradoxo da Tartaruga de Zenão


Zenão de Eleia, filósofo grego pré-socrático, pertencia
à escola eleática, cujo pensamento era o de negação do
movimento, com a justificativa de que, se um corpo
está parado, logo tem um estado de ser, e se ele
adquire movimento, ele tem um estado de negação do
Figura 15: Esquematização do trem 1 sendo ultrapassado pelo trem 2. O ser anterior; e, como algo não pode ser e não-ser ao
ponto vermelho serve de referência para enxergar a ultrapassagem mesmo tempo, então os membros da escola eleática
negavam o movimento.
Para que o trem 1 complete a ultrapassagem sobre o Para exemplificar essa pretensiosa posição, Zenão
2 ele deve percorrer a mesma distância que o trem 2 propôs o seguinte problema: suponha que Aquiles,
percorreu, além de percorrer o comprimento do trem 2 e herói da mitologia grega, conhecido pela sua astúcia,
depois percorrer o próprio comprimento. Para ficar mais dispute corrida com uma tartaruga. Para conferir
fácil de visualizar, observe o ponto vermelho em cada uma
vantagem a esta, ela inicia seu movimento mais à
das etapas de ultrapassagem. Em termos de velocidade
frente, posição b; enquanto Aquiles inicia em a. Após o
relativas, isto é, o quanto o trem 1 andou a mais que 2
(logo, é como se a velocidade do trem 2 fosse nula): início da disputa, quando Aquiles chega em b, a
1) O trem 1 alcança o trem 2 tartaruga está em c, quando Aquiles atinge c, a
2) O trem 1percorre o comprimento do trem 2 tartaruga está em d, e assim sucessivamente. Com isso,
3) O trem 1 percorre a distância equivalente ao próprio Zenão explica que não há ultrapassagem, portanto,
comprimento. não há movimento.
Com isso, a igualdade da função horária ficará da
seguinte forma:
𝐬𝟐 = 𝐬𝟏 + 𝐥𝟏 + 𝐥𝟐 (𝟒)
Em que l1 e l2 são os comprimentos do trem 1 e 2,
respectivamente.

Tipos de movimento no gráfico


Agora que já foi entendido como representar
graficamente as funções horárias, vamos enxergar como
analisar o tipo de movimento (progressivo ou retrógrado)
a partir do gráfico.
Nas figuras 13 e 14, as retas têm inclinações
positivas, isto é, são inclinadas para cima. Se a inclinação
é positiva, a velocidade também o é. Sendo a velocidade A partir da figura acima, fica mais claro entender o
positiva, isso significa que o movimento é progressivo. porquê do argumento de Zenão para a
Portanto, se a inclinação é para cima (positiva), igual as impossibilidade do movimento estar incorreta.
duas imagens acima, o movimento é progressivo. Mas, Perceba que Aquiles se aproxima cada vez mais da
ATENÇÃO! Essa consideração é válida para o gráfico s x t. tartaruga, ou seja, o intervalo de tempo cd é menor
Se o gráfico for v x t, basta que a reta seja horizontal que bc, que por sua vez, é menor que ab. Desse modo,
(estamos tratando somente de velocidade constante. Mais
se levar em conta o tempo corrido, Aquiles ultrapassa
à frente veremos casos em que a velocidade também
a tartaruga. Apesar de ser óbvio saber que há
varia) e na parte positiva do eixo da velocidade para que
o movimento seja progressivo. Para o movimento ultrapassagem, o enunciado de Zenão é muito bem
elaborado para chegar ao seu ponto de vista.

13
Eluxidando a Física

Aceleração
Assim como a velocidade é uma variação Ora, se houve variação da velocidade, pela
temporal do espaço (ou a variação do espaço em relação definição descrita acima de aceleração, pode-se dizer que
à variação do tempo), a aceleração é uma variação houve uma aceleração? A resposta é SIM! A aceleração é
temporal da velocidade. Pode ser entendida também uma variação da velocidade. Para isso, temos que
como o ritmo de aumento ou de diminuição da velocidade diferenciar os tipos de aceleração:
em um dado intervalo de tempo. Dada a definição, a Aceleração escalar: quando houve variação da
velocidade é dada matematicamente como: velocidade escalar com o tempo. No exemplo acima,
∆𝐯 𝐯𝐟 − 𝐯𝐢 conclui-se que não houve aceleração escalar, pois a
𝐚= = (𝟓) velocidade escalar final é igual à inicial.
∆𝐭 𝐭 𝐟 − 𝐭 𝐢
Aceleração vetorial: se trata da aceleração no seu
Em conteúdo de escola e vestibular, as grandezas
sentido mais completo, isto é, não leva em conta somente
físicas param de variar na aceleração. Quando se tem
o seu valor, mas a direção e o sentido dela também. No
aceleração em 99% dos casos a aceleração é constante
exemplo acima, a intensidade da velocidade não variou,
(quando não é constante, o exercício fornece um gráfico,
mas a direção e o sentido sim, levando a uma aceleração.
e o exercício é todo baseado em análise gráfica). Uma
A aceleração é, então, resultado da soma de duas
aceleração constante leva a uma velocidade variada. É
componentes de aceleração: a componente tangencial,
totalmente análogo ao caso da velocidade constante em
que é a responsável pela variação do módulo da
relação ao espaço que também varia.
velocidade (aceleração tangencial), e aceleração
A unidade da velocidade no SI (Sistema
centrípeta, responsável, pela mudança na direção do
Internacional de Unidades) é m/s. A aceleração, como é
vetor velocidade (aceleração centrípeta), mas que não
velocidade dividida pelo tempo. Logo, a unidade da
muda o módulo desta.
aceleração é:
m  Aceleração tangencial (a T ): muda o módulo da
m 1 m velocidade.
[a] = s = . =
s s s s²  Aceleração centrípeta (a C ): muda a direção e o sentido
Isto é, metros por segundo a cada segundo. Se da velocidade.
uma aceleração é de +1 m/s², isso significa que a
velocidade varia de +1 m/s a cada segundo.
Aceleração vetorial
Assim como a aceleração escalar é a variação da
velocidade escalar no intervalo de tempo em que ocorreu
essa variação da velocidade, a aceleração vetorial é a
variação da velocidade vetorial no intervalo de tempo em
que houve essa variação vetorial da velocidade.
∆𝐯⃗⃗
𝐚⃗⃗ = (𝟔) Figura 17: a aceleração e suas componentes de um corpo realizando
∆𝐭 uma curva e acelerando ao mesmo tempo.
Se o que é procurado é o módulo da aceleração
vetorial:
Exemplo 7: Um móvel descreve uma circunferência de
|Δv⃗⃗| Δv
|a⃗⃗| = ou a = raio 10 m, completando a volta a cada 10 segundos.
Δt Δt Determine, para um intervalo de 5 segundos de
Se é dito que um carro entrou em um túnel a 100
movimento, o módulo da velocidade vetorial e da
km/h e na saída estava nos mesmos 100 km/h. Podemos
aceleração vetorial.
afirmar que houve variação de velocidade? Cuidado, pois
essa é uma pergunta delicada. A resposta é: depende. Se
Resposta: Para descobrir a velocidade escalar do móvel, é
estiver falando do módulo da velocidade (ou velocidade
necessário calcular a distância percorrida em uma volta
escalar), de fato não houve mudança de velocidade, pois o
da circunferência:
valor se manteve a 100 km/h, ou seja, não houve
C = 2. π. r
aceleração escalar. Mas, vale lembrar que velocidade é C = 2. π. 10 = 20π m
uma grandeza vetorial, ou seja, além do módulo, tem
Como o móvel percorre essa distância em 10
direção e sentido. Se houve uma variação na direção e
segundos, a velocidade escalar é:
sentido, então pode-se dizer que houve uma variação da 20π
velocidade. De modo geral: v= = 2π m/s
10
 Velocidade escalar: não variou! Se em 10 segundos completa uma volta, em 5
 Velocidade vetorial: variou! segundos completa metade da volta, conforme o
Isso pode ser mostrado quando se desenha os esquema a seguir:
vetores. Fazendo uma soma vetorial, percebe-se que
houve uma variação.

Figura 16: a velocidade na horizontal para direitas antes, e vertical para


baixo depois. Sendo um vetor, a velocidade mudou.

14
Eluxidando a Física

A variação do deslocamento do móvel foi


exatamente o valor do diâmetro (ou duas vezes o valor Figura 18: condições em que ocorrem um movimento acelerado. Em
do raio) da circunferência: ambos os casos a velocidade e aceleração apontam para a mesma
direção.
|r⃗| = 2.10 = 20 m
O módulo da velocidade vetorial fica:
|r⃗| 20 m Movimento retardado
|v
⃗⃗| = = = 4 m/s Esse movimento pode ocorrer de duas formas: 1)
Δt 5s
O módulo da aceleração vetorial é: o móvel está se deslocando com velocidade positiva
|Δv
⃗⃗| (movimento progressivo, ou seja, no sentido positivo do
|a⃗⃗| = eixo dos espaços) e a aceleração é negativa ou 2) o móvel
Δt
⃗⃗1 = +2π m/s
v está se deslocando com velocidade negativa (movimento
⃗⃗2 = −2π m/s
v retrógrado, ou seja, no sentido negativo do eixo dos
Obs.: o sinal negativo indica que o vetor está para espaços) e a aceleração é positiva. Muito importante ficar
esquerda, pois está contrário ao que foi orientado como atento ao fato de que não importa o sinal da aceleração, e
positivo. sim a discordância do seu sinal com o sinal da velocidade.
Δv ⃗⃗ = v ⃗⃗2 − v ⃗⃗1 = −2π − (+2π) Nesse movimento, se a velocidade e a aceleração têm
Δv ⃗⃗ = −4π m/s sentidos contrários, o corpo retarda seu movimento.
|Δv ⃗⃗| = 4π m/s
|Δv ⃗⃗| 4π
|a⃗⃗| = = = 0,8π m/s²
Δt 5
Obs.: se a pergunta fosse em torno da velocidade
escalar, a distância deveria ser o comprimento do arco em
vez do deslocamento.

Aceleração centrípeta
A aceleração centrípeta, conforme já foi falado, é
a aceleração que não altera o módulo da velocidade, mas
sim muda a direção do corpo. Essa aceleração é calculada
pela seguinte equação:
𝐯² Figura 19: condições em que ocorrem um movimento retardado. Em
𝐚𝐜 = (𝟕)
𝐫 ambos os casos a velocidade e aceleração apontam para direções
contrárias.
Movimento Uniformemente Variado (MRUV ou MUV)
Um movimento progressivo retardado é aquele
Agora analisaremos o tipo de movimento
no qual a velocidade ainda está positiva, apesar de
havendo aceleração. Por que o movimento acelerado é
diminuindo. Por exemplo, se em 2016 o Brasil cresceu
chamado de uniformemente variado? Isso se deve à
7%, em 2017, 4% e, em 2018, cresceu 2%, ele ainda está
velocidade crescer linearmente, uma vez que a aceleração
em crescimento ainda que esteja diminuindo.
é constante. Então, quais são os movimentos no MUV? São
os seguintes:
Classificação dos movimentos no gráfico (velocidade x
Movimento acelerado
tempo)
Esse movimento pode ocorrer de duas formas: 1)
o móvel está se deslocando com velocidade positiva Primeiro vamos analisar o caso da velocidade
(movimento progressivo, ou seja, no sentido positivo do pelo tempo (v x t) de um corpo. Os exercícios mais comuns
eixo dos espaços) e a aceleração também é positiva ou 2) do MUV são gráficos em que a velocidade vai variando e o
o móvel está se deslocando com velocidade negativa movimento deve ser analisado como progressivo ou
(movimento retrógrado, ou seja, no sentido negativo do retrógrado e acelerado ou retardado. Vamos fazer isso
eixo dos espaços) e a aceleração também é negativa. para o seguinte caso:
Muito importante ficar atento ao fato de que não importa
o sinal da aceleração, e sim a concordância do seu sinal
com o sinal da velocidade.

15
Eluxidando a Física

Trecho 1:
 v < 0 (velocidade negativa) → movimento Figura 20 : gráfico v x t com velocidade constante, isto é,
retrógrado sem aceleração.
 a < 0 (velocidade fica cada vez mais negativa;
inclinação para baixo) Para calcular o espaço percorrido por um móvel
 Movimento retrógrado acelerado (segundo caso que tem o movimento de acordo com o gráfico, podemos
da Figura 18) – velocidade e aceleração no ter alguns raciocínios:
mesmo sentido - O móvel percorre 10 m/s (10 metros a cada
Trecho 2: segundo) durante 5 segundos. Logo, em 5 segundos ele
 v < 0 (velocidade negativa) → movimento andou 50 m;
∆s ∆s
retrógrado - v = ∆t → 10 = 5−0 → ∆s = 50 m
 a = 0 (velocidade não muda. Está constante) → Não importa qual dos dois raciocínios foram
movimento uniforme utilizados, a resposta é a mesma, 50 m., que é exatamente
 Movimento retrógrado uniforme – não há a área embaixo do gráfico! E isso faz todo o sentido. Para
aceleração, portanto não se classifica em entender o porquê faz sentido, vamos fazer uma análise
acelerado ou retardado. dimensional:
Trecho 3: m
 v < 0 (velocidade negativa) → movimento [A] = b. h = s. = m
s
retrógrado A área do gráfico tem exatamente a unidade de
 a > 0 (inclinação para cima) espaço. Agora vejamos o seguinte gráfico:
 Movimento retrógrado retardado (segundo caso
da Figura 19) – velocidade e aceleração em
sentidos contrários
Trecho 4:
 v = 0 (em cima do eixo do tempo)
 a > 0 (inclinação para cima)
 Movimento acelerado – como não há velocidade,
mas tem aceleração, o movimento é acelerado,
necessariamente
Trecho 5:
 v>0 (velocidade positiva) → movimento
progressivo
 a > 0 (inclinação para cima)
 Movimento progressivo acelerado (primeiro Figura 21: gráfico v x t com velocidade crescendo linearmente, isto é,
aceleração constante.
caso da Figura 18) – velocidade e aceleração no
mesmo sentido
Pede-se para calcular a aceleração da figura,
Trecho 6:
pode-se usar a definição, equação (5). Basta ver o quanto
 v > 0 (velocidade positiva) → movimento
varia da velocidade em relação ao tempo. Uma maneira
progressivo
gráfica é basicamente calcular o coeficiente angular da
 a=0
reta (um pouco mais à frente, em Bônus Equação Primeiro
 Movimento progressivo uniforme
e Segundo Grau abordará sobre equação da reta). Quanto
Trecho 7:
mais inclinada a reta, maior a aceleração. Uma reta
 v > 0 (velocidade positiva) → movimento
horizontal tem coeficiente angular nulo, logo, não há
progressivo
aceleração.
 a < 0 (inclinação para baixo)
Num gráfico s x t a área embaixo do gráfico não
 Movimento progressivo retardado (primeiro
tem significado físico. Mas a inclinação da reta fornece a
caso da Figura 19) – velocidade e aceleração em
velocidade, assim como a inclinação do gráfico v x t
sentidos contrários.
fornece a aceleração.
Cálculo da Velocidade e Espaço no Gráfico
Um gráfico pode fornecer muito mais informação
do que somente análise de movimento. Veja a figura a
seguir:

16
Eluxidando a Física

Equações do MUV
Outro tópico de extrema importância dentro da
cinemática são as equações para o cálculo da velocidade e
do espaço percorrido no MUV.
Cálculo da velocidade a partir da definição de
aceleração:
∆v vf − vi
a= = → a. t = vf − vi → vf = vi + a. t
∆t t
𝐯 = 𝐯𝟎 + 𝐚. 𝐭 (𝟖)
Assim como foi feito para a equação (3), a
equação (8) pode ser representada de modo semelhante
à figura (12):
A aceleração na queda livre é g, com valor
aproximado de 10 m/s². A velocidade em qualquer
instante de tempo é dada por (velocidade inicial nula):
∆v v − v0 v
a= = → g = → v = gt
∆t t − t0 t
Figura 22: representação visual da equação (4) veja que quanto maior o Representando graficamente a velocidade pelo
tempo e a aceleração, maior será o vetor a.t pelo fato de que o veículo
variará mais intensamente a velocidade e por mais tempo. tempo:

Cálculo da velocidade (sem utilizar o tempo) –


Equação de Torricelli:
𝐯 𝟐 = 𝐯𝟎𝟐 + 𝟐. 𝐚. 𝚫𝐬 (𝟗)
Obs.: a equação não será demonstrada
pois envolve conceitos de energia e trabalho, tópico do
próximo e-book.
Cálculo do espaço:
Sabemos, do movimento uniforme, que a equação
do espaço é:
s = s0 + v. t
A área do gráfico representa o espaço percorrido
No entanto, aqui no MUV, a velocidade é variável. pelo corpo (nesse caso, a altura em que inicou a queda
Então precisamos calcular a velocidade média de um livre, dado que gt é a velocidade final):
trecho qualquer que o móvel percorre de modo que esteja
acelerando: gt × t gt 2
v + v0 h= →h=
vm = 2 2
2
No MUV, a velocidade média é a média das De modo geral:
velocidades inicial e final. at 2
s=
Agora vamos escrever a velocidade final em 2
função da velocidade inicial usando a equação (6),
Para que a equação fique completa, é necessário
definição da aceleração. Logo, a equação acima ficará:
o termo da velocidade inicial e do espaço inicial somando
v + v0 (v0 + a. t) + v0 2. v0 + a. t ao último termo inserido acima, chamado de proporção de
vm = = = Galileu.
2 2 2
a. t a. t a. t²
vm = v0 + → vm = v0 + s = s0 + v 0 t +
2 2 2
Usando essa equação da velocidade média na As equações (8), (9) e (10) são as principais
equação do espaço, temos, por fim (ufa!): equações do movimento uniformemente variado. Com
a. t a. t elas, é possível resolver equações tanto pensando em
s = s0 + vm . t = s0 + (v0 + ) . t = s 0 + v0 . t + .t movimentos acelerados na horizontal, quanto
2 2 movimentos verticais, como queda de corpos, uma vez
𝐚. 𝐭 𝟐 que no lugar da aceleração a, podemos colocar a
𝐬 = 𝐬𝟎 + 𝐯𝟎 . 𝐭 + (𝟏𝟎)
𝟐 aceleração da gravidade, g.
Agora, vamos olhar dois exemplos que mostram,
Outra forma de chegar na equação (10) é de modo simples, como aplicar essas equações mostradas
observar o comportamento de um corpo submetido à acima. Os exemplos são ilustrações simples. Exercícios
aceleração da gravidade. Durante a queda a sua mais complicados estão na lista de exercícios ao final
velocidade é dada conforme o desenho abaixo. deste e-book.

Exemplo 8: Um corpo é abandonado, a partir do repouso,


a uma altura de 20 metros. Sabendo que g=10 m/s²,

17
Eluxidando a Física

calcule a velocidade final que o corpo atinge ao tocar o m/s. Com o tempo é possível ir até mesmo memorizando
solo, em km/h. Desconsidere a resistência do ar. algumas conversões.
Portanto, a velocidade com que o objeto toca ao
Resposta: Para um exercício que não fornece figura, solo é de 72 km/h.
reforço que é interessante fazer uma figura ilustrativa de
modo a compreender melhor o exercício. Exemplo 9: Um automóvel está trafegando a uma
velocidade de 108 km/h na rodovia, quando percebe que,
a 50 metros de distância tem um trânsito parado à frente.
Sabendo que o veículo parou completamente após 4
segundos, qual foi sua aceleração?

Resposta: Segue a ilustração abaixo:

É conveniente, também, arbitrar um sentido


positivo para o seu sistema. Com isso, fica mais fácil saber
se os sinais do espaço, da velocidade e da aceleração.
Os dados são: v0 = 0, h = 20 m e g = 10 m/s²
Calcular: v = ? Nesse caso, o exercício forneceu o tempo, então
Note que em nenhum momento o exercício falou devemos fazer uso da equação do espaço no MUV:
sobre tempo. Se não tem tempo, usar a equação de a. t 2
Torricelli. s = s0 + v 0 . t +
2
v 2 = v02 + 2. a. d → v 2 = 02 + 2. g. h = 2.10.20 Note que tem o termo s0 . Nesse caso podemos
v 2 = 400 → v = 20 m/s colocar o veículo, no instante em que começou a frear
Logo, a velocidade do corpo, ao tocar o solo é de (isto é, o instante t=0), no espaço inicial, s0 = 0.
20 m/s. Outro ponto importante é que sua velocidade
Mas a velocidade que o exercício pediu foi em inicial era de 108 km/h. Como vamos utilizar no SI,
km/h. Se a resposta for dada em m/s estará errada. A devemos fazer a conversão de km/h para m/s. A partir da
conversão é feita da seguinte maneira: tabela, temos que v0 = 30 m/s.
Como o espaço inicial foi dado como nulo, então no
momento que o veículo para, ele fica no espaço s=50 m.
Agora que temos todas as informações necessárias para o
cálculo da aceleração, vamos resolvê-la:
a. 42
50 = 0 + 30.4 + → 50 = 120 + 8. a → 8. a = −70
2
𝐚 = −𝟖, 𝟕𝟓 𝐦/𝐬²

BÔNUS: Equação do primeiro e do segundo grau


Multiplicar por 3,6 em alguns casos pode dar
números com casas decimais e contas chatas. Mas tem As equações do primeiro e segundo grau são
alguns números que aparecem com uma frequência muito largamente utilizadas para o movimento uniforme e para
alta. A tabela abaixo expõe as principais conversões: o movimento uniformemente variado para o cálculo do
deslocamento e da velocidade. Muitas vezes são utilizados
como sinônimos os termos equação e função (acabo
Em destaque, a conversão mais importante de km/h para usando como sinônimos, coloquialmente, neste e-book),
m/s: mas na verdade a equação é um ponto de interesse da
km/h m/s função, como, por exemplo, quando iguala a função a 0. O
valor da função (denominada de f(x) ou de y, ambos são a
3,6 1
mesma coisa) depende especificamente da variável x,
5,4 1,5
enquanto os outros parâmetros da função são constantes.
7,2 2
10,8 3 Função do primeiro grau
36 10 Função do primeiro grau é toda função cujo grau
54 15 da variável x é 1, conforme está escrito abaixo:
72 20
90 25 f(x) = ax + b
108 30 𝐲 = 𝐚𝐱 + 𝐛 (𝟏𝟏)

Esses são os principais valores usados. Pode Toda função do primeiro grau é uma reta, em que
aparecer valores como 900 km/h. Se você souber que 90 a e b são parâmetros a serem encontrados dessa função,
km/h é 25 m/s, basta multiplicar por 10, o que dá 250 de modo que os valores desses parâmetros permitem
conhecer o comportamento da reta. Os dois parâmetros

18
Eluxidando a Física

da função do primeiro grau fornecem duas características Note que, diferente do caso anterior, a variação
da reta: inclinação da reta e ponto em que a reta corta o dos valores de y em relação a x é o mesmo (a diferença
eixo y. dos valores de y1 e de y1 sempre ficam constantes), o que
Inclinação da reta: A inclinação é dada pela já era esperado de enxergar antes mesmo de ver a tabela,
relação do quanto varia os valores de y em relação a x. Por uma vez que ambos têm o mesmo coeficiente angular,
exemplo, vejamos os dois seguintes casos: a=2, por isso a variação de ambos, em relação ao x, é 2.
y1 = 2x Mas veja que as duas funções, apesar de variarem igual,
y2 = 5x partem de valores diferentes de y. E este valor em x=0 é
A tabela a seguir mostra como y1 e y2 variam para justamente o parâmetro b. Verifique abaixo:
os mesmos valores de x. y = ax + b → y(0) = a. 0 + b
y(0) = b
x=1 x=2 x=3 x=4 Exemplo 10: Encontre a equação da reta que
y1 2 4 6 8 passa pelos pontos (4,5) e (6,11).
y2 5 10 15 20 Resposta: Para isso, o jeito mais braçal de
Graficamente: encontrar a equação da reta é substituir os dois pontos na
equação da reta (y = ax + b) e resolver o sistema linear
que resulta disso:
5 = a. 4 + b
{
11 = a. 5 + 11
Entretanto, há um jeito mais lógico e rápido,
aproveitando o que foi exposto dos parâmetros acima:
Δy 11 − 5 6
a= = = =3
Δx 6−4 2
A equação da reta fica da seguinte maneira:
y = 3x + b
Para encontrar b, basta substituir qualquer um
dos dois pontos, (4,5) ou (6,11), na equação acima:
(4,5)
y = 3x + b → 5 = 3.4 + b → b = 5 − 12 = −7
y = 3x − 7
Verifique que substituindo por (6,11) o resultado
seria o mesmo, isto é, b=-7.
Esse método de resolução se aplica a qualquer
Figura 23: representação das retas e as respectivas inclinações. equação do primeiro grau. Portanto, esqueça métodos
para decorar a equação da reta. Tudo o que foi passado
Podemos, portanto, afirmar que a inclinação está até aqui é o suficiente.
diretamente relacionada à variação de y em relação em x.
Logo: Equação do segundo grau
Função do segundo grau é toda função cujo grau
∆𝐲 da variável x é 2, conforme está escrito abaixo:
𝐚= (𝟏𝟐)
∆𝐱
f(x) = ax 2 + bx + c
Agora, será investigado o parâmetro b. Verifique 𝐲 = 𝐚𝐱 𝟐 + 𝐛𝐱 + 𝐜 (𝟏𝟑)
os casos abaixo:
y1 = 2x + 1 Para que este tópico não se estenda muito, os
y2 = 2x + 5 parâmetros a, b e c serão explicados rapidamente, uma
A tabela abaixo mostra como ficam os valores de vez que o principal ponto é explicar sobre métodos de
y1 e y2 variam conforme x: resolução das equações do segundo grau.

x=0 x=1 x=2 x=3


y1 1 3 5 7
y2 5 7 9 11

Figura 24: alguns comportamentos possíveis da função do segundo grau.


Encontrar a equação dessa função é encontrar os pontos em que y=0,
isto é, os pontos em que a função corta o eixo x.
 a:
o a > 0: concavidade para cima
o a > 0: concavidade para baixo

É comum exercícios que envolvem equação do


segundo grau, como no caso do MRUV. Para isso, será
apresentado três modos de resolver uma equação do
 c:
segundo grau:
o c > 0: corta o eixo y na parte positiva
1) Bhaskara
o c < 0: corta o eixo y na parte negativa
Método mais conhecido para se resolver uma
equação do segundo grau. Consiste em duas etapas:

19
Eluxidando a Física

y = ax 2 + bx + c Ok, você pode estar achando que complicou


∆= b2 − 4ac demais a questão. Sugiro treinar em pelo menos quatro
−b ± √∆ exercícios. Se ainda assim achar muito complicado, aí
x= pode descartar essa opção.
2a
−b + √∆ −b − √∆ Utilize esse método nas seguintes equações 5
x1 = e x2 = 2
a) y = x + 4x + 3
2a 2a
Em se tratando do método mais usado, não será b) y = x 2 + 8x + 12
necessário fazer exemplos. c) y = x 2 − 4x + 4
2) Soma e Produto d) y = x 2 + 10x + 21
Método também apresentado aos estudantes,
mas bem menos usado. Consiste em descobrir o x por Movimento Circular Uniforme (MCU)
tentativa e erro. Com a prática, algumas equações passam O MCU se trata de um movimento muito análogo
a ficar muito simples de enxergar as raízes. ao movimento retilíneo uniforme, visto no início deste e-
y = ax 2 + bx + c book. Entretanto, as letras usadas serão diferentes, uma
b vez que não estamos mais falando de distância linear dada
x1 + x2 = − em m, km, mas sim de distância angular, dada em graus
a
c ou radianos.
x1 . x2 =
a Apesar de se tratar de outro tópico, que é o
Exemplo 11: Calcular as raízes da função movimento circular, as equações não serão provadas
f(x)=x²+6x+8 porque a analogia com o movimento retilíneo é direta, isto
Resposta: Conforme acima: é, as equações são as mesmas. Só mudam as letras.
6 Antes de estabelecer esse paralelo entre os
x1 + x2 = − = −6
1 movimentos retilíneo e circular, há algumas definições no
8 MCU que são importantes antes de ir propriamente às
x1 . x2 = = 8
1 equações.
A pergunta a se fazer é: quais dois números que Todo movimento circular em algum momento
somados dão -6 e multiplicados dão 8? Esses números são retorna ao seu ponto inicial, uma vez que esse movimento
-4 e -2. Veja como se encaixam. Esse método é muito mais se constitui num ciclo. E sempre que há um ciclo, faz-se
rápido para calcular as raízes se colocada em prática. No necessário definir frequência.
início demanda mais tempo.
3) Fatoração
Temida por muita gente, a fatoração consiste em
transformar a equação num modo mais simplificado a
partir dos produtos notáveis. São exemplos:
 Diferença de quadrados: a2 − b2 = (a + b)(a − b)
 Operação distributiva: a(b + c) = a. b + a. c
 Quadrado perfeito: (a + b)2 = a2 + 2ab + b²
 Etc.
Vamos ver como aplicaremos a fatoração num
exemplo.
f(x) = x 2 + 6x + 8 → x 2 + 6x + 8 = 0
Olhando apenas os dois termos, x²+6x podemos
Figura 25: Tudo que funciona por meio de ciclos, seja um processador,
forçar a conseguir um quadrado perfeito com isso do cujo processamento funciona por ciclos, seja algo componente que gira,
seguinte modo: tem frequência.
(x + 3)2 = x 2 + 6x + 9 A grosso modo, frequência mensura quantas
(x + 3)2 − 9 = x 2 + 6x vezes um ciclo (no caso do movimento circular, um ciclo é
Veja que foi escolhido o (x+3)² justamente uma volta) é completado a cada unidade de tempo. Por
porque o os dois primeiros termos são x² e 6x, mas o 9 é exemplo: 3 voltas por segundo, 10 ciclos por minuto, etc.
somado, e devemos compensar isso na fatoração. Fica do A Terra tem um movimento circular em torno do seu
seguinte modo: próprio eixo, logo, podemos calcular a frequência desse
x 2 + 6x + 8 = 0 movimento. Como o dia tem 24 horas, então a Terra
(x + 3)2 − 9 + 8 = 0 completa seu ciclo nesse tempo. Para calcular a
Como o quadrado perfeito faz aparecer o número frequência, em número de voltas por hora, temos:
9, é necessário que esse 9 seja subtraído. 1 volta
(x + 3)2 − 1 = 0 f= = 0,042 volta/h
24 h
(x + 3)2 = 1 Se quisermos, em vez de saber o número de voltas por
x + 3 = √1 hora, mas quantos graus por hora, basta lembrar que uma
x + 3 = ±1 volta corresponde a 360°:
x + 3 = ±1 360°
f= = 15°/h
x = −3 ± 1 24 h
x1 = −3 − 1 = −4 Além da frequência, outra importante definição é
x2 = −3 + 1 = −2 a do período. O período tem a definição inversa da

5Respostas: a) -1 e -3; b) -2 e -6; c) raiz


dupla 2; d) -3 e -7

20
Eluxidando a Física

frequência: mensura quantas vezes um período de tempo A velocidade angular tem relação com a
é necessário para completar um ciclo. Não somente a frequência. Mas a primeira é calculada em rad/s (radianos
definição é inversa, como a própria equação que relaciona por segundo), enquanto a segunda é calcula em Hz (ciclos
ambos também é: por segundo).
1 1 É sabido que uma volta corresponde a 2π
f = ou T =
T f radianos. Logo, uma frequência de 1 Hz (1 volta por
1 −1 segundo) corresponde a 2π rad/s. Com isso, a equação
[ f ] = = s = Hz (Hertz)
s que relaciona frequência e velocidade angular é:
Lembrete: 𝛚 = 𝟐. 𝛑. 𝐟 (𝟏𝟖)
Frequência: espaço angular num dado tempo
Período: um dado espaço angular no tempo Para aplicar essas equações nos exercícios,
seguem dois exemplos.
Espaço Angular
Exemplo 12: Um corpo, em movimento circular
O espaço angular, conforme já foi dito, tem uniforme, parte de um ângulo de 30° do eixo horizontal e
grande semelhança com o espaço retilíneo. se movimenta a uma velocidade de 120° por segundo.
 Movimento retilíneo uniforme: 𝐬 = 𝐬𝟎 + 𝐯. 𝐭 (𝟑)
Quantas voltas completas o corpo terá dado após 15
 Movimento circular uniforme: 𝛉 = 𝛉𝟎 + 𝛚. 𝐭 (𝟏𝟒) segundos?
Em que 𝜃 (letra do alfabeto grego chamada de Resposta: Substituindo os valores, temos:
theta, lê-se “téta”), expresso em radianos, é espaço
angular final, θ0 é o espaço angular inicial (também
expresso em radianos) e ω é a velocidade angular, que no
MCU é uniforme, ou seja, não varia.
Assim como feito para o caso linear, no caso
angular a seguinte frase é válida: "o ponto que eu saio
mais o ponto que eu ando é o ponto que eu chego".
Além dessa analogia entre o deslocamento linear
e angular, há uma relação direta entre elas, dada por:

𝐬 = 𝛉. 𝐫 (𝟏𝟓)

Em que s é o comprimento do arco que


corresponde à variação angular.
Associando as informações do enunciado às
variáveis da equação, temo:
θ0 = 30°, f = 120°/s e t = 15s
Vamos colocar tudo em unidades do SI, em rad e
rad/s.
360° − 2π rad
30° − θ0
30.2. π π
θ0 = =
360 6
360°/s − 2π rad/s
120°/s − ω
120.2. π 2π
ω= = rad/s
360 3
Figura 26: exemplo de um movimento circular uniforme genérico. Agora, vamos aplicá-las na equação:

Velocidade Angular π 2π π 30. π π


θ = θ0 + ω. t = + . 15 = + = + 10π
A definição da velocidade angular, ω (letra do 6 3 6 3 6
π
alfabeto grego chamada de ômega), pode ser obtida de θ = + 10π
modo análogo à definição da velocidade linear, v. 6
Como cada volta tem 2π rad, então foram dadas 5
𝚫𝛉 voltas completas, além de fazer um arco de 30°. Portanto,
𝛚= (𝟏𝟔)
𝚫𝐭 foram dadas 5 voltas completas.
Obs. 1: o exercício poderia ter sido feito sem fazer
Além da analogia entre velocidade linear e
a conversão para radianos. Só foi feita a conversão para
angular, a equação (17) mostra a relação que há entre
ver como é o procedimento. Mas, cuidado! Nem todos
ambas:
exercícios podem usar em graus.
𝐯 = 𝛚. 𝐫 (𝟏𝟕) Exemplo 13: Dois corpos, A e B, estão se
movimentando sobre uma circunferência. O corpo A parte
do espaço angular -30° a uma velocidade angular horária

21
Eluxidando a Física

de 90°/s, enquanto B parte do ângulo 60° a uma Logo, fica válida a relação:
velocidade angular anti-horária de π rad/s. Em qual
instante os corpos A e B se encontram? v1 = v2
Resposta: esse exercício é muito semelhante ao 𝛚𝟏 . 𝐫𝟏 = 𝛚𝟐 . 𝐫𝟐 (𝟏𝟗)
exemplo 4. Antes de fazer a função horária de cada um, é
necessário deixar todos na mesma unidade. Será optado
pelo uso em rad e rad/s.

Figura 27: dois elementos rotativos giram com velocidade angular


diferente nos dois casos, mas no caso da esquerda giram em sentidos
contrários, enquanto o da direita giram no mesmo sentido.

Os dois casos apresentados foram de velocidades


lineares iguais. Mas há um caso em que as velocidades
angulares podem ser iguais: se os dois elementos
estiverem presos num mesmo eixo, como na figura
Corpo A: abaixo:
𝜋
ωA = −90°/s = − rad/s
2
O sinal da velocidade angular de A está negativa
porque o positivo foi colocado como no sentido anti-
horário. Poderia ter colocado a referência no sentido
horário.
11π
θ0A = −30° = 330° = rad
12

A equação do movimento angular será:


θA = θ0A + ωA . t
11π π
θA = − .t Figura 28: dois elementos rotativos acoplados pelo mesmo eixo. Nesse
12 2 caso ambos têm a mesma velocidade angular.
Corpo B:
ωB = π rad/s ω1 = ω2
π 𝐯𝟏 𝐯𝟐
θ0B = 60° = rad = (𝟐𝟎)
3 𝐫𝟏 𝐫𝟐
A equação do movimento angular será:
θB = θ0B + ωB . t
π Mecânica da Bicicleta
θB = + π. t Na bicicleta, a catraca é menor que a coroa. Sendo
3
Como os corpos devem se encontrar, teremos a a catraca menor, ela gira com velocidade angular maior
relação de igualdade entre os espaços angulares: para compensar a mesma velocidade linear da coroa. A
11π π π 11π π π roda traseira da bicicleta está acoplada à catraca.
θA = θB → − . t = + π. t → − . t = + π. t Portanto, o conjunto catraca-coroa giram à mesma
12 2 3 12 2 3
velocidade linear enquanto o conjunto catraca-roda giram
11 1 1 11 1 1 7 3 à mesma velocidade angular quando se está pedalando.
→ − . t = + 1. t → − = t+t→ = .t
12 2 3 12 3 2 12 2 Mas, como o raio da roda é muito maior que o da catraca,
𝟕 a velocidade linear da roda é muito maior, o que faz a
𝐭= 𝒔 bicicleta atingir uma velocidade considerável. Se a
𝟏𝟖
velocidade linear fosse a mesma da catraca, a bicicleta não
Elementos rotativos acoplados andaria mais rápido que uma pessoa caminhando
Tem dois casos comuns de elementos rotativos tranquilamente.
(elementos que possuem velocidade angular) acoplados: O que a marcha de uma bicicleta possibilita é
ligados por contato direto e ligados por meio de uma mudar a corrente de posição tanto da coroa quanto da
correia. catraca. Quando a corrente vai para um raio menor da
Contato direto: quando duas engrenagens estão catraca e maior da coroa aumenta-se a relação de raios
encaixadas diretamente uma na outra, pode-se afirmar entre a coroa e a catraca, logo, a catraca gira com uma
que na região de contato, tanto uma engrenagem quanto velocidade angular ainda maior, aumentando a
a outra, possuem a mesma velocidade linear. Uma velocidade angular da roda e, portanto, a velocidade
engrenagem terá velocidade no sentido horário e outra no linear da mesma o que leva a bicicleta a uma maior
sentido anti-horário. velocidade. Portanto, quando a marcha fica mais “pesada”,
Ligados por correia: quando duas polias são a corrente vai para um maior raio da coroa e menor da
ligadas por uma correia, ambas também possuem a catraca. Pode reparar isso quando for andar de bicicleta.
mesma velocidade linear. No entanto giram no mesmo
sentido.

22
Eluxidando a Física

Exemplo 14: A figura abaixo mostra quatro As equações do MCUV são as apresentadas a
engrenagens vistas de cima. Sabendo que não há nenhum seguir (colocadas junto com as equações do MUV para
deslizamento entre elas e que a engrenagem 1 gira a 2 ficar fácil fazer a analogia):
Hz, calcule:
a) a velocidade linear da engrenagem 4, se r4 = Deslocamento angular:
(1/π) m. 𝐚. 𝐭 𝟐
b) a velocidade angular da engrenagem 4. 𝐬 = 𝐬𝟎 + 𝐯𝟎 . 𝐭 + (𝟏𝟎)
𝟐
c) O sentido de giro de cada uma das engrenagens. 𝛂. 𝐭²
𝛉 = 𝛉𝟎 + 𝛚𝟎 . 𝐭 + (𝟐𝟏)
𝟐
Velocidade angular:
𝐯 = 𝐯𝟎 + 𝐚. 𝐭 (𝟖)
𝛚 = 𝛚𝟎 + 𝛂. 𝐭 (𝟐𝟐)

𝐯 𝟐 = 𝐯𝟎𝟐 + 𝟐. 𝐚. 𝚫𝐬 (𝟗)
𝛚𝟐 = 𝛚𝟐𝟎 + 𝟐. 𝛂. 𝚫𝛉 (𝟐𝟑)

Resposta: as engrenagens estão conectadas de Relação entre aceleração linear e angular:


diferentes maneiras. Há casos em que a conexão é por 𝐬 = 𝛉. 𝐫 (𝟏𝟓)
contato (entre 1 e 2 e entre 3 e 4), e há caso em que a 𝐯 = 𝛚. 𝐫 (𝟏𝟕)
conexão é por eixo (entre 2 e 3). 𝐚 = 𝛂. 𝐫 (𝟐𝟒)
a)
v1 = v2 Exemplo 15: Um disco de raio de 0,5 m inicia seu
ω1 . r1 = ω2 . r2 movimento com uma aceleração de 20°/s². No momento
2. π. f1 . r1 = 2. π. f2 . r2 em que atinge uma velocidade de π rad/s, inicia um
2.1,5r = f2 . 3r processo de desaceleração no qual para completamente
3 = 3. f2 em 3 s. Determine:
f2 = 1 Hz a) Qual a aceleração linear do disco até atingir a
ω2 = ω3 velocidade máxima.
2. π. f2 = 2. π. f3 b) Em quanto tempo atinge a velocidade máxima.
f2 = f3 c) O deslocamento linear total da ponta do disco.
f3 = 1 Hz d) A aceleração centrípeta no instante t=8s.

v3 = v4 Resposta: O corpo tem MCUV e, por isso, as


ω3 . r3 = ω4 . r4 equações a serem usadas serão as apresentadas acima.
2. π. f3 . r3 = 2. π. f4 . r4 a) A aceleração linear é dada pela relação da equação
f3 . r3 = f4 . r4 (24). Mas lembre-se que a unidade da aceleração angular
1.1,5r = f4 . 2r deve ser passada para o SI, isto é, rad/s²
1,5
f4 = = 0,75 Hz 180°/s² − π rad/s²
2
1 20°/s 2 – α
v4 = ω4 . r4 = 2π. f4 . r4 = 2π. 0,75. = 1,5 m/s
π 20. π π
α= = rad/s²
3π 180 9
b) ω4 = 2. π. f4 = 2. π. 0,75 = 2 rad/s
π π 1 π
c) Como a engrenagem 1 está em sentido horário: a = α. r = . 0,5 = . = m/s²
Engrenagem 2: sentido anti-horário 9 9 2 18
3,14
Engrenagem 3: sentido anti-horário a≈ m/s² = 0,17 m/s²
Engrenagem 4: sentido horário. 18
O resultado poderia ter ficado em função de π. A
Esse exercício exemplifica o funcionamento de
substituição de π por 3,14 foi somente para ter uma ideia
uma caixa de câmbio de um carro.
do valor da aceleração.
b) ωmáx. = π rad/s
Movimento Circular Uniformemente Variado(MCUV)
A melhor equação a ser utilizada, uma vez que o
Assim como o MCU está para o MU, o MCUV está tempo é pedido, é (22):
para o MUV. As equações a serem utilizadas do MCUV ω = ω0 + α. t
serão tomadas a partir das equações do MUV. π
Esse tópico será explicado de forma breve, uma π=0+ .t
18
vez que esse conteúdo é pouco recorrente tanto nas π
π=0+ .t
escolas quanto em vestibulares, mas ainda assim é bom 18
tomar conhecimento. 1
1= . t → t = 18 s
O MCUV é um caso em que o corpo tem um 18
movimento puramente rotativo, mas diferente do MCU, c) Para o deslocamento linear total, é necessário calcular
essa rotação é acelerada (aceleração esta que é constante), a desaceleração do disco após atingir a velocidade
isto é, a velocidade angular (assim como a linear) varia. máxima. Para esse caso, a velocidade final será zero, e a
inicial será π rad/s.
ω = ω0 + α. t

23
Eluxidando a Física

0 = π + α. 3
π v² (ω. r)² ω2 . r²
α2 = − rad/s² ac = = = = ω2 . r
3 r r r
Deslocamento do primeiro trecho: 𝜋 2 π² π²
ω2 = ω20 + 2. α. Δθ ac = ( ) . r = . 0,5 = m/s²
𝜋 9 81 162
π2 = 0² + 2. . Δθ a c = 0,062 m/s²
18 Obs.: π²≅10.
𝜋
π2 = 2. . Δθ
18
Δθ = θ = 9π rad Lançamento Oblíquo
𝑠 = 𝜃. 𝑟 = 9𝜋. 0,5 O lançamento oblíquo é todo lançamento (v0 ≠
𝑠 = 4,5π m 0) cujo ângulo do corpo lançado com a horizontal é um
Outro modo de pensar (muito mais matemático ângulo tal que esteja no intervalo 0 ≤ θ ≤ 90°.
do que físico), uma vez que obteve 9π de deslocamento Nesse lançamento há uma composição de
angular, é o seguinte: movimento do corpo no eixo x e no eixo y que têm
2π rad − 1 volta comportamentos distintos. No eixo y, isto é, na vertical, o
9π rad − n voltas corpo sofre a ação da aceleração da gravidade. Se sofre a
2π. n = 9π ação de uma aceleração, as equações são do MUV. Já o eixo
9π 9 x, isto é, eixo horizontal, não há ação de nenhuma
n= = = 4,5 voltas
2π 2 aceleração ou ação externa6, com isso, as equações que se
Agora, deve ser lembrado que o comprimento de dão nessa componente são todas do MU.
uma volta é 2πr: O lançamento oblíquo que é arremessado do solo
1 volta − 2πr forma uma parábola (figura 27), devido à ação da
4,5 voltas − s aceleração da gravidade. Logo, as propriedades e
s. 1 = 4,5.2. π. r características vistas para exercícios de parábola são
s = 9π. r = 9π. 0,5 usados no lançamento oblíquo em seu movimento no eixo
9π y. A equação do espaço do MUV é uma função do segundo
𝑠1 = m
2 grau, que, conforme vimos no tópico bônus das equações
Esse modo foi mais trabalhoso. Mas foi colocado de primeiro e segundo grau, é uma parábola.
como uma outra alternativa. Sempre é bom enxergar uma Mas muito cuidado! São duas parábolas que
resolução por alternativas diferentes, ainda que uma das surgem em lançamento oblíquo: 1) A parábola que o
resoluções seja pior. O importante é ter o conhecimento movimento todo do lançamento oblíquo descreve no ar,
porque pode ter casos em que a segunda alternativa é conforme a figura 27 (ou seja, está em função do espaço)
mais rápida. e; 2) A parábola que o movimento vertical descreve num
Agora, na desaceleração foi fornecido o tempo. movimento de subida, de parada na altura máxima, e de
Logo, a melhor equação a ser utilizada é a (21): descida (está em função do tempo).
α. t²
θ = θ0 + ω0 . t +
2
𝜋
− . 3²
θ = 0 + π. 3 + 3
2

θ = 3π −
6

θ = 3π −
2
18𝜋 9π
θ= −
6 6
18π − 9π 9π 3π
θ= = =
6 6 2
s = θ. r Figura 29: movimento parabólico do lançamento oblíquo. Note que a
3𝜋 velocidade horizontal é sempre igual. A velocidade na vertical varia, mas
s= . 0,5 nos pontos simétricos da parábola o tamanho do vetor é o mesmo. Note
2 que a referência para o sinal positivo é quando está apontado para cima.
3π A gravidade, nesse caso, será negativa devido ao referencial. Poderia ter
s2 = m optado pelo referencial para baixo. (Fonte:
4
Agora basta somar s1 e s2 : https://phet.colorado.edu/sims/html/projectile-
9π 3π 18π 3π 21π motion/latest/projectile-motion_en.html)
s1 + s2 = + = + = m
2 4 4 4 4
d) Para calcular a aceleração centrípeta, basta saber qual Agora, serão apresentadas as equações
a velocidade linear do disco no instante t=2 s: referentes ao lançamento oblíquo. Em nenhuma delas
ω = ω0 + α. t será colocado o número da equação, pois não se trata de
π nenhuma equação nova. Todas foram retiradas do MU e
ω = 0+ .2 do MUV. Portanto, não decore as equações que estão
9
2π abaixo, só lembre de relacioná-las às equações já
ω= rad/s conhecidas.
9

6Na verdade, em um caso real, um corpo sofre ação no eixo x também, vestibulares e de conteúdo de ensino médio a resistência do ar é
que é a força de resistência do ar. Portanto, a rigor, as equações do MUV desprezada.
deveriam ser usadas para ambos os eixos. Entretanto, para fins de

24
Eluxidando a Física

Equações do movimento na horizontal: 2. v0 . sen(θ) v02 . 2. sen(θ). cos(θ)


 Velocidade: A = v0 . cos(θ). =
g g
vx
sen(θ) = → vx = v0 . sen(θ) Pela equação do arco duplo:
v0 sen(2θ) = 2. sen(θ). cos(θ)
 Alcance (distância percorrida na horizontal): Portanto, temos que:
∆s A
vx = = → A = vx . t = v0 . sen(θ). 𝑡 𝐯𝟎𝟐 . 𝐬𝐞𝐧(𝟐𝛉)
∆t t 𝐀= (𝟐𝟐)
Equações do movimento na vertical: 𝐠
 Velocidade: Como o maior valor possível para a função seno é
vy 1, e o ângulo que fornece esse valor de seno é 90°, temos:
cos(θ) = → vy = v0 . cos(θ) 2θ = 90°
v0
vy = v0y + a. t = v0 . sen(θ) + a. t 𝛉 = 𝟒𝟓°
A menor velocidade de todo o trajeto parabólico
vy = v02y + 2. g. h (Equação de Torricelli)
2
que o corpo percorre é justamente no ponto de altura
Para o tempo de subida, sabendo que na altura máxima, uma vez que há apenas a componente x da
máxima a velocidade do corpo, na componente y, é zero velocidade, que é inalterada. Em todo o resto do trajeto,
(isso não significa que o corpo esteja parado, pois a além da componente x há a componente y que varia. Por
componente vx segue sempre com o mesmo valor. menor que seja a componente y ao longo do trajeto (num
valor diferente de zero), isso significa que a velocidade
 Altura: total é sempre maior que a velocidade de cada
O corpo atinge a altura máxima exatamente componente. Mas, quando está no ponto mais alto, a
quando completa seu ponto de subida. velocidade total é a própria velocidade do componente x.
a. t 2 g. t 2 Por isso a menor velocidade ocorre no ponto mais alto da
s = s 0 + v0 . t + → h = h 0 + v0 y . t −
2 2 trajetória.
Para o caso da altura máxima, o corpo atinge essa Agora, vejamos com dois exemplos a aplicação
altura máxima exatamente quando completa seu tempo dessas equações:
de subida. (o sinal do termo da gravidade está negativo
por se tratar da subida do corpo, que é contrário ao Exemplo 16: Um corpo é lançado com velocidade
sentido da gravidade. O v0y e g devem ter sinais contrários inicial igual a 50m/s, a 30° com a horizontal. Desprezando
num lançamento oblíquo cuja velocidade inicial é para o atrito e considerando g = 10m/s², determine o alcance
cima). máximo horizontal desse corpo.
g. t sub 2 Resposta: Segue o esboço do descrito no
hmáx. = h0 + v0y . t sub − enunciado:
2
Para o caso mais tradicional, que o corpo é
lançado de uma altura inicial, h0 , nula, e o corpo termina
o movimento também na altura nula, o tempo de subida é
igual ao tempo de descida. Isso significa que o tempo de
subida é metade do tempo total.
Vale lembrar que como forma um triângulo retângulo,
basta fazer o Teorema de Pitágoras para fazer a relação
entre v0 e suas componentes:
v02 = vx2 + vy2 → v0 = √vx2 + vy2
A = vx . t = v0 . cos(θ). t = 50. cos(30°) . t
Essas equações apresentadas resolvem 90% dos Agora, basta calcular o tempo necessário para o
problemas que envolvem lançamento oblíquo. Veja que corpo atingir a altura máxima. Esse tempo encontrado
não foi utilizado nenhuma fórmula ou equação para ser será multiplicado por 2 para encontrar o tempo total do
decorada. São equações que você já conhece! movimento oblíquo.
Em alguns casos pede-se para provar que o vy = v0y − g. t
ângulo que leva ao alcance máximo do corpo, isto é, a Na altura máxima a velocidade vertical do corpo
distância horizontal, é o ângulo de lançamento de 45°. Isso é zero.
será provado logo em seguida: 0 = v0 . sen(θ) − g. t
∆s A 0 = 50. sen(30°) − 10. t sub
vx = = → A = vx . t = v0 . cos(θ). t (𝐚)
∆t t 10. t sub = 50. sen(30°)
Para o movimento de subida: 50. sen(30°) 50.0,5
vy = v0y + g. t 0 t sub = =
10 10
0 = v0 . sen(θ) − g. t sub t sub = 2,5 s
v0 . sen(θ) t = 2. t sub = 2.2,5
t sub =
g t= 5s
t v0 . sen(θ) Agora, retornando para a equação do alcance:
= A = 50. cos(30°) . t
2 g
2. v0 . sen(θ) √3
t= (𝐛) A = 50. .5
g 2
Substituindo a equação (b) na equação (a), temos A = 125√3 m

25
Eluxidando a Física

Exemplo 17: Da janela de um prédio, a 20 m do chão, é Como uma componente é muito maior que a
arremessada uma pedra horizontalmente, de forma a outra, o valor da hipotenusa será muito próximo ao valor
tocar no chão a 5 m da base do prédio, conforme esquema da maior componente. Isso é uma boa aproximação.
a seguir. Considerando g=10 m/s², calcule: Comparando a resposta dada com a que leva em conta os
a) O tempo que a pedra demora, desde o seu lançamento, 2,5², há um erro menor que 1%.
para tocar no chão. a) Para descobrir qual a função da parábola em função do
b) A velocidade inicial da pedra ao ser arremessada. espaço, basta enxergar o eixo vertical como eixo y e o
c) A velocidade da pedra ao atingir o chão. horizontal como eixo x.
d) A equação da trajetória da pedra y=f(x). y = ax 2 + bx + c
Resposta: O desenho esquemático da situação Quando a pedra é solta, x=0 e y=0 (referencial
encontra-se a seguir (veja que está arbitrado aonde é o adotado)
referencial e em que sentido ele cresce): 0 = a. 02 + b. 0 + c
c=0
Quando a pedra cai no chão, x=5 e y=20
y = ax 2 + bx
20 = a. 52 + b. 5
20 = 25a + 5b → 4 = 5a + b
Como a parábola é totalmente simétrica em
relação ao seu ponto de vértice, temos que o outro ponto
em que a pedra toca no solo é quando x=-5 m (lembre-se
que pensar num x negativo é um artifício matemático para
encontrar a função):
y = ax 2 + bx
20 = a. (−5)2 + b. (−5)
20 = 25a − 5b
4 = 5a − b
Para encontrar a e b, usa-se um sistema:
a) O cálculo do tempo pode ser feito a partir do eixo 5a + b = 4
{
vertical do lançamento oblíquo. Como o tempo é pedido, a 5a − b = 4
equação a ser usada será do deslocamento vertical: Vamos ver uma maneira lógica de encontrar a
a. t² resposta: veja que nos dois casos tem 5a e 4. Isso significa
h = h0 + v0 y . t + que mesmo somando ou substraindo b a resposta não
2
A altura final é de 20 metros, enquanto a inicial é de 0. muda. Isso significa que b=0
Como o sentido arbitrado é favorável ao sentido da 5a + 0 = 4
gravidade, então a aceleração será 10 m/s² positiva. 4
a = = 0,8
Além disso, a pedra não tem velocidade inicial na 5
componente vertical, v0y : Portanto:
y = f(x) = 0,8x²
10. t²
20 = 0 + 0. t + Agora uma pergunta teórica: qual objeto cai mais
2 rápido? Um que é abandonado do repouso ou um que tem
10. t²
20 = uma velocidade inicial para a direita, conforme ilustra a
2 figura(19)?
40 = 10. t²
4 = 1. t²
𝐭 = 𝟐𝐬
b) Como o tempo de queda é de 2 segundos e a pedra se
move 5 metros na horizontal, velocidade esta que é
constante, temos:
∆s 5
vx = =
∆t 2
𝐯𝐱 = 𝟐, 𝟓 𝐦/𝐬
c) Quando o enunciado pede a velocidade da pedra, sem
se referir qual componente, o pedido é a velocidade total,
isto é, o valor que é encontrado fazendo Teorema de Figura 30: qual dos dois casos o corpo cai primeiro?
Pitágoras com as componentes.
v 2 = vx2 + vy2 Na verdade, apesar de induzir a responder que o
A velocidade vx é constante o tempo todo, logo corpo da esquerda cairia primeiro, os dois corpos cairão
juntos, pois o que importa é a velocidade na vertical.
basta encontrar vy . Este valor pode ser encontrado com a
g. t 2
seguinte equação: h = h0 + v 0 y . t −
vy = v0y + g. t 2
O tempo de queda do corpo está ligado somente
vy = 0 + 10.2 à altura inicial, à velocidade inicial em y e à gravidade.
vy = 20 m/s Para todos esses 3 parâmetros, ambos são iguais.
v 2 = (2,5)² + 20² Portanto, tocarão no solo ao mesmo tempo.
𝐯 ≈ 𝟐𝟎 𝐦/𝐬

26
Eluxidando a Física

Exercícios devido a obras na pista. Supondo que o carro freie a uma


E1) A partir do gráfico s x t (espaço x tempo) seguinte, aceleração constante, calcule o tempo necessário até a
indique qual a velocidade do móvel nos instantes 1s, 3s, sua parada e a aceleração para que pare na distância
5s, 7s e 9s. (Note que o gráfico fornece o espaço e é pedido necessária.
a velocidade. Nesse caso, deve-se fazer uso da equação da
velocidade) E7) A figura abaixo contém a seguinte disposição das
engrenagens. Sabendo que, r1 = 10 cm, r2 = 25 cm, r3 =
15 cm, r4 = 25 cm, r5 = 30 cm e r6 = 5 cm, e que f1 =
60 rpm, calcule a velocidade angular da engrenagem 6 e a
relação da velocidade angular entre 1 e 6.

E2) Um móvel 1 com uma velocidade de 5 m/s no sentido


positivo do eixo dos espaços parte 15 metros à frente de
um móvel 2 que está a uma velocidade de 8 m/s.
Considerando que ambos percorrem o mesmo sentido do
eixo. Em quanto tempo os móveis se encontrarão? E em
qual ponto do eixo dos espaços? E8) (UFU) Uma pessoa dispõe de um motor que gira a
5.000 rpm e acopla-o, usando correias que não
E3) Calcule a velocidade média do móvel nos 20
escorregam, a três polias (1, 2 e 3), de modo a buscar
segundos.
novas configurações de velocidade e de rotação,
diferentes das que o motor proporciona. A, B e C são três
pontos marcados nas extremidades das polias 1, 2 e 3,
respectivamente. Considere, também, que Ra > Rb > Rc.
A figura a seguir representa o acoplamento realizado.

E4) um móvel 1 com uma velocidade de 5 m/s no sentido


positivo do eixo dos espaços parte 30 metros à frente de
um móvel 2 que está a uma velocidade de 8 m/s.
Considerando que os móveis percorrem sentido oposto
do eixo dos espaços. Em quanto tempo os móveis se
encontrarão? E em qual ponto do eixo dos espaços?

E5) Calcule o módulo da variação vetorial da velocidade


no seguinte caso: Quando acionado o motor, a relação entre as velocidades
(V) verificadas nos pontos A, B e C e o número de rotações
por minuto (RPM) de cada polia é:

a) VA > VB > VC e RPM1 = RPM2 = RPM3

b) VA = VB = VC e RPM1 < RPM2 < RPM3

c) VA = VB > VC e RPM1 < RPM2 < RPM3

d) VA < VB < VC e RPM1 < RPM2 < RPM3

E6) Um automóvel está trafegando numa via a 72 km/h, E9) Dois corpos, A e B, estão se movimentando sobre uma
quando observa carros parados 20 metros à frente circunferência. A parte do espaço angular -30° a uma

27
Eluxidando a Física

velocidade angular horária de π rad/s, enquanto B parte a) os módulos das velocidades angulares 𝜔𝑅 da roda
do ângulo 60° a uma velocidade angular anti-horária de dianteira da bicicleta e 𝜔𝐷 do eixo do dínamo, em rad/s;
π/2 rad/s 1 segundo após o A partir. Em qual instante os b) o tempo T que o eixo do dínamo leva para completar
corpos A e B se encontram pela quarta vez? Nesse uma volta;
instante, quantas voltas completas A deu? E B?
Note e adote: π=3. Considere que o contato do eixo do
E10) (Fuvest) Arnaldo e Batista disputam uma corrida de dínamo com o pneu se dá em R = 50 cm.
longa distância. O gráfico das velocidades dos dois atletas,
no primeiro minuto da corrida, é mostrado a seguir: E13) (Fuvest – 2011) Uma menina, segurando uma bola
de tênis, corre com velocidade constante, de módulo igual
a 10,8 km/h, em trajetória retilínea, numa quadra plana e
horizontal. Num certo instante, a menina, com o braço
esticado horizontalmente ao lado do corpo, sem alterar o
seu estado de movimento, solta a bola, que leva 0,5 s para
atingir o solo. As distâncias sm e sb percorridas,
respectivamente, pela menina e pela bola, na direção
horizontal, entre o instante em que a menina soltou a bola
(t = 0 s) e o instante t = 0,5 s, valem:

a) sm = 1,25 m e sb = 0 m.
b) sm = 1,25 m e sb = 1,50 m.
a) A aceleração a B de Batista em t=10 s; c) sm = 1,50 m e sb = 0 m
b) As distâncias dA e dB percorridas por Arnaldo e Batista, d) sm = 1,50 m e sb = 1,25 m
respectivamente, até t= 50 s; e) sm = 1,50 m e sb = 1,50 m.
c) A velocidade média vA de Arnaldo no intervalo de
tempo entre 0 e 50 s. E14) (Unicamp 2018) Esteiras rolantes horizontais são
E11) (Fuvest - Modificada) Um DJ, ao preparar seu frequentemente instaladas em grandes aeroportos para
equipamento, esquece uma caixa de fósforos sobre o disco facilitar o deslocamento das pessoas em longos
de vinil, em um toca-discos desligado. A caixa se encontra corredores. A figura ao lado mostra duas esteiras rolantes
a 10 cm do disco. Quando o toca-discos é ligado, no que se deslocam em sentidos opostos com velocidades
instante t=0, ele passa a girar com aceleração angular constantes em relação ao piso em repouso (𝑣⃗𝑒1 e 𝑣⃗𝑒2 ) e de
constante 𝛼= 1,1 rad/s², até que o disco atinja a mesmo módulo, igual a 1,0 m/s. Em um mesmo instante,
frequência final f=33 rpm que permanece constante. duas pessoas (representadas por A e B) que se
Determine: deslocavam com velocidade constante de módulo igual a
vA = 1,5 m/s e vB = 0,5 m/s em relação ao piso e em
a) A velocidade angular final do disco, ωf , em rad/s. sentidos contrários entram nas esteiras e continuam
b) O instante t f em que o disco atinge a velocidade caminhando como anteriormente, como mostra a figura.
angular ωf . As esteiras rolantes têm comprimento total de 120 m.
Note e adote: Aceleração da gravidade local g=10 m/s²;
π=3.

E12) (Fuvest – Modificada) Um ciclista pedala sua


bicicleta, cujas rodas completam uma volta a cada 0,5
segundo. Em contato com a lateral do pneu dianteiro da
bicicleta, está o eixo de um dínamo que alimenta uma
lâmpada, conforme a figura ao lado. Os raios da roda
dianteira da bicicleta e do eixo do dínamo são,
respectivamente, R = 50 cm e r = 0,8 cm. Determine:

a) Calcule o tempo necessário para que a pessoa A chegue


até a outra extremidade da esteira rolante.
b) Quanto tempo depois de entrarem nas esteiras as
pessoas A e B passam uma pela outra?

E15) (Unicamp – 2017) Em 2016 foi batido o recorde de


voo ininterrupto mais longo da história. O avião Solar
Impulse 2, movido a energia solar, percorreu quase 6480

28
Eluxidando a Física

km em aproximadamente 5 dias, partindo de Nagoya no do vento é v=18 km/h, teria uma frequência de rotação
Japão até o Havaí nos Estados Unidos da América. A de
velocidade escalar média desenvolvida pelo avião foi de
aproximadamente
a) 54 km/h
b) 15 km/h
c) 1296 km/h
d) 6480 km/h

E16) (Unicamp – 2017) O semáforo é um dos recursos


utilizados para organizar o tráfego de veículos e de
pedestres nas grandes cidades. Considere que um carro
trafega em um trecho de uma via retilínea, em que temos
3 semáforos. O gráfico abaixo mostra a velocidade do
carro, em função do tempo, ao passar por esse trecho em
que o carro teve que parar nos três semáforos. A distância a) 3 rpm
entre o primeiro e o terceiro semáforo é de b) 200 rpm
c) 720 rpm
d) 1200 rpm
Se necessário, π≈3.

E19) Encontre a velocidade linear na superfície da terra


na latitude de 30°, sabendo que a velocidade linear na
linha do Equador é 1675 km/h.

E20) (Unicamp - Modificada) Recentemente, a sonda New


Horizons tornou-se a primeira espaçonave a sobrevoar
Plutão, proporcionando imagens espetaculares desse
astro distante. A sonda saiu da Terra em janeiro de 2006
e chegou a Plutão em julho de 2015. Considere que a
a) 330 m sonda percorreu uma distância de 4,5 bilhões de
b) 440 m quilômetros nesse percurso e que 1 ano é
c) 150 m aproximadamente 3x107 s. Calcule a velocidade escalar
d) 180 m média da sonda nesse percurso, em km/s.
E17) (Unicamp – 2016) A demanda por trens de alta E21) (Unicamp – 2015) Considerando que a massa e as
velocidade tem crescido em todo o mundo. Uma dimensões dessa estrela são comparáveis às da Terra,
preocupação importante no projeto desses trens é o espera-se que a aceleração da gravidade que atua em
conforto dos passageiros durante a aceleração. Sendo corpos próximos à superfície de ambos os astros seja
assim, considere que, em uma viagem de trem de alta constante e de valor não muito diferente. Suponha que um
velocidade, a aceleração experimentada pelos corpo abandonado, a partir do repouso, de uma altura h =
passageiros foi limitada a amax = 0,09g, onde g=10 m/s2 54 m da superfície da estrela, apresente um tempo de
é a aceleração da gravidade. Se o trem acelera a partir do queda t = 3,0 s. Desta forma, pode-se afirmar que a
repouso com aceleração constante igual a amax, a aceleração da gravidade na estrela é de
distância mínima percorrida pelo trem para atingir uma a) 8 m/s²
velocidade de 1080 km/h corresponde a b) 10 m/s²
a) 10 km c) 12 m/s²
b) 20 km d) 18 m/s²
c) 50 km
d) 100 km E22) (Unicamp) Considere um computador que
armazena informações em um disco rígido que gira a uma
E18) (Unicamp – 2016) Anemômetros são instrumentos frequência de 120 Hz. Cada unidade de informação ocupa
usados para medir a velocidade do vento. A sua um comprimento físico de 0,2 m na direção do
construção mais conhecida é a proposta por Robinson em movimento de rotação do disco. Quantas informações
1846, que consiste em um rotor com quatro conchas magnéticas passam, por segundo, pela cabeça de leitura,
hemisféricas presas por hastes, conforme figura abaixo. se ela estiver posicionada a 3 cm do centro de seu eixo,
Em um anemômetro de Robinson ideal, a velocidade do como mostra o esquema simplificado apresentado
vento é dada pela velocidade linear das conchas. Um abaixo? Considere π≈3.
anemômetro em que a distância entre as conchas e o
centro de rotação é r=25 cm, em um dia cuja velocidade

29
Eluxidando a Física

I) Desprezando a resistência do ar:


a. no ponto de altura máxima, a velocidade do projétil é
nula.
a) 1,62 x 106 b. o projétil chega a B com velocidade nula.
b) 1,8 x 106 c. a velocidade vetorial do projétil ao atingir B é igual à
c) 64,8 x 108 de lançamento.
d) 1,08 x 108 d. durante o movimento, há conservação das
componentes horizontal e vertical da velocidade.
E23) (Unicamp) As máquinas cortadeiras e colheitadeiras e. durante o movimento, apenas a componente
de cana-de-açúcar podem substituir dezenas de horizontal da velocidade é conservada.
trabalhadores rurais, o que pode alterar de forma II) Quanto ao módulo da velocidade, tem valor
significativa a relação de trabalho nas lavouras de cana- mínimo igual a:
de-açúcar. A pá cortadeira da máquina ilustrada na figura a. 125 m/s
abaixo gira em movimento circular uniforme a uma b. 100 m/s
frequência de 300 rpm. A velocidade de um ponto c. 75 m/s
extremo P da pá vale (Considere π≈3) d. zero
e. 25 m/s

E26) Se um pequeno furo horizontal for feito na parede


vertical de um reservatório que contenha um líquido
ideal (sem viscosidade), um filete líquido escoará pelo
furo, e sua velocidade inicial terá intensidade v = √2gh,
onde g é o módulo da aceleração da gravidade. Considere
o movimento do fluido como o de um projétil lançado no
vácuo, a partir do furo, com velocidade v ⃗⃗. Podemos
afirmar que a distância horizontal, L, percorrida pelo
fluido é:
a) 9 m/s
(H − h)v
b) 15 m/s a)
c) 18 m/s g
d) 60 m/s
b) √−4h2 + 4Hh
4(H − h)
E24) Um menino, andando de skate com velocidade v = c)
v
2,5 m/s num plano horizontal, lança para cima uma
d) √2vg
bolinha de gude com velocidade v0 = 4,0 m/s e a apanha
de volta. Considere g = 10 m/s². (H − h)v
a) A trajetória descrita pela bolinha em relação à Terra. e) √
2g
b) A trajetória descrita pela bolinha em relação ao
menino.
E27) Uma bola chutada horizontalmente de cima de uma
c) A altura máxima que a bolinha atinge.
laje, com velocidade V0, tem sua trajetória parcialmente
d) A distância horizontal que a bolinha percorre.
registrada em uma foto, representada no desenho abaixo.
e) O valor da velocidade da bolinha, em relação ao solo,
A bola bate no chão, no ponto A, voltando a atingir o chão
quando ela atinge a altura máxima.
em B, em choques parcialmente inelásticos.
E25) Um projétil é lançado em certa direção com
velocidade inicial v
⃗⃗0 , cujas projeções vertical e horizontal
têm módulos, respectivamente, de 100 m/s e 75 m/s. A
trajetória descrita é parabólica e o projétil toca o solo
horizontal em B.

30
Eluxidando a Física

E30) A posição de uma partícula depende do tempo de


acordo com x=t²-5t+1. Calcule:

a) O deslocamento e a velocidade média no intervalo t=3s


até t=4s
b) A fórmula geral do deslocamento no intervalo de t até
t+Δt.
c) A velocidade instantânea em qualquer instante t.

E31) O maquinista de um trem de passageiros viajando a


30 m/s avista um trem de carga a 200 m à frente, na
mesma linha. O trem de carga está viajando no mesmo
sentido com uma velocidade de 10 m/s. O maquinista do
trem de passageiros imediatamente aplica os freios,
causando uma desaceleração de 1 m/s², enquanto o trem
a) Estime o tempo T, em s, que a bola leva até atingir o de carga continua com a mesma velocidade constante.
chão, no ponto A. a) Haverá colisão?
b) Calcule a distância D, em metros, entre os pontos A e B. b) Se existir, onde se dará?
c) Determine o módulo da velocidade vertical da bola vA ,
em m/s, logo após seu impacto com o chão no ponto A. E32) Um foguete para pesquisas meteorológicas é
lançado verticalmente para cima. O combustível, que lhe
Note e adote: Nos choques, a velocidade horizontal da imprime uma aceleração de 1,5g (g=aceleração da
bola não é alterada. Desconsidere a resistência do ar, o gravidade) durante o período de queima, esgota-se após
atrito e os efeitos de rotação da bola. 0,5 min.

E28) A figura a seguir mostra dois discos de papelão a) Qual seria a altitude máxima atingida pelo foguete, se
fixados a um mesmo eixo, que gira com frequência igual a pudéssemos desprezar a resistência do ar?
50 Hz. Os discos foram fixados a locais distantes 2 m um b) Com que velocidade, e depois de quanto tempo, ele
do outro. Um projétil foi disparado paralelamente ao eixo, voltaria a atingir o solo?
movendo-se em um movimento suposto retilíneo e
uniforme, perfurando dois discos. O ângulo entre o o E33) (AFA 2018) O gráfico seguinte representa a
plano que contém o eixo e o furo no primeiro disco e o velocidade escalar v de uma partícula em movimento
plano que contém o eixo e o furo do segundo disco é igual retilíneo.
a 45°. Determine a velocidade do projétil.

Considerando que, em t = 0, a partícula está na origem dos


espaços (S0 = 0), o gráfico que melhor representa a
posição (S) dessa partícula até o instante t = 5 s é
E29) Dois discos de raios R A = 4 cm e R B = 40 cm,
perpendiculares entre si, giram sem deslizar um sobre o
outro, como mostra a figura. A distância entre o ponto de
contato do disco A e o centro do disco B é x e a velocidade
angular do disco B é de 20 rad/s. Qual a expressão da
velocidade angular ω do disco A em função de x (medido
em cm)?
a)

b)

31
Eluxidando a Física

b) 160 m
c) 2,5 m/s
E11)
a) ωf =3,3 rad/s
b) 𝑡𝑓 = 3 𝑠
E12)
a) ωR = 12 rad/s
c) b) ωD = 750 rad/s
E13) E
E14) a) 48s
b) 30s
E15) A
E16) A
d)
E17) C
E34) (IME 2019) Uma partícula desloca-se solidária a um E18) B
trilho circular com 0,5 m de raio. Sabe-se que o ângulo q, E19) 15,8 km/s
indicado na figura, segue a equação θ = t² , onde t é o E20) C
tempo em segundos e q é o ângulo em radianos. O módulo E21) D
do vetor aceleração da partícula, em t = 1 s , é: E22) C
E23)
a) Arco de parábola
b) Reta
c) 0,8 m
d) 2,0 m
e) 2,5 m/s
E24)
I) E
II) C
E25) C
E26)
a) 0,8 s
b) 2,4 m
a) √5
c) 6 m/s
b) √2
E27) 800 m/s
c) 1
E28) 5x
d) 2√5 E29)
e) 2 a) 2 m e 2 m/s
b) Δx=2t.Δt-5.Δt+(Δt)²
Gabarito c) v=2t-5
E1) E30)
t=1 s: 5 m/s a) Sim
t=3 s: 0 b) Após 400m ao iniciar a freada.
t=5 s: -10 m/s E31)
t=7 s: 0 a) 16,5 km
t=9 s: 10 m/s b) 570 m/s e 133 s
E2) t=5s e s=40 m E32)
E3) 11,375 m/s a) 3375 m
E4) t=10 s
b) v = 150√3 e t = 15√3
s=80 m
E33) D
E5) 50 km/h
E34) A
E6) t=2 s e a=-10 m/s²
E7) ω6 = 2,88π rad/s
𝜔6
= 1,44
𝜔1
E8) B
E9) 16/3 s. Corpo A: 2 voltas, Corpo B: 1 volta
Dica: Comece |θB | + |θA | = n. 2π, em que n é a n-ésima
vez em que os corpos se encontram. Pode ser também
θB − θA = n. 2π.
E10)
a) 0,2 m/s²

32

Você também pode gostar